You are on page 1of 63

Total Marks : 200

Online Prelims TEST - 16 (SUBJECT WISE)


( InsightsIAS Mock Test Series for UPSC Preliminary Exam 2020 ) Mark Scored : 0

1 Which of the following was/were the views of early nationalists on labour class movements
1. They did support the factory acts of 1881 and 1891.
2. They differentiated between the labour in the Indian owned factories and those in the British-owned
factories.
3. They were indifferent to the labour’s cause.

Select the correct answer using the code given below


A. 1 and 2 only
B. 2 and 3 only
C. 3 only
D. None

Correct Answer : B

Answer Justification :

The early nationalists, especially the Moderates,

● were indifferent to the labour’s cause; Hence Statement 3 is correct.

● differentiated between the labour in the Indianowned factories and those in the British-
owned factories; Hence Statement 2 is correct.

● believed that labour legislations would affect the competitive edge enjoyed by the Indian-owned
industries;

● did not want a division in the movement on the basis of classes;

● did not support the Factory Acts of 1881 and 1891. Hence Statement 1 is incorrect.

2 Consider the following statements


1. Bipin Chandra Pal and G. Subramanya Aiyar demanded better conditions for workers.
2. Sasipada Banerjea started the newspaper called Deenbandhu

Which of the statements given above is/are correct?


A. 1 only
B. 2 only
C. Both 1 and 2
D. Neither 1 nor 2

Correct Answer : A

Answer Justification :

There were many prominent nationalist leaders like Bipin Chandra Pal and G. Subramanya
Aiyar who demanded better conditions for workers and other pro-labour reforms. Hence

1
Total Marks : 200
Online Prelims TEST - 16 (SUBJECT WISE)
( InsightsIAS Mock Test Series for UPSC Preliminary Exam 2020 ) Mark Scored : 0

Statement 1 is correct.

Narain Meghajee Lokhanday started the newspaper Deenbandhu and set up the Bombay Mill
and Millhands Association. Hence Statement 2 is incorrect.

3 Which of the following personalities was/were involved in labour strikes during Swadeshi Upsurge?
1. Chidambaram Pillai
2. Ashwini Coomar Banerjea
3. S.A. Dange

Select the correct answer using the code given below


A. 3 only
B. 1 only
C. 1 and 2 only
D. None

Correct Answer : C

Answer Justification :

Workers participated in wider political issues during Swadeshi Upsurge. Strikes were
organised by Ashwini Coomar Banerjea, Prabhat Kumar Roy Chaudhuri, Premtosh Bose and
Apurba Kumar Ghosh.

These strikes were organised in government press, railways and the jute industry.

The whole of 1928 witnessed unprecedented industrial unrest. This period also saw the
crystallisation of various communist groups, with leaders like S.A. Dange, Muzaffar Ahmed, P.C.
Joshi, Sohan Singh Joshi etc.

4 Consider the following statements


1. All India Trade Union Congress (AITUC) was established in late 1930’s.
2. Lala Lajpat Rai was elected as the first president of AITUC
3. Lajpat Rai was the first to link capitalism with imperialism

Which of the statements given above is/are correct?


A. 2 only
B. 3 only
C. 1 and 3 only
D. 2 and 3 only

Correct Answer : D

Answer Justification :

2
Total Marks : 200
Online Prelims TEST - 16 (SUBJECT WISE)
( InsightsIAS Mock Test Series for UPSC Preliminary Exam 2020 ) Mark Scored : 0

The All India Trade Union Congress was founded on October 31, 1920. Hence Statement 1 is
incorrect.

The Indian National Congress president for the year, Lala Lajpat Rai, was elected as the first
president of AITUC and Dewan Chaman Lal as the first general secretary.

Lajpat Rai was the first to link capitalism with imperialism— “imperialism and militarism are
the twin children of capitalism”.

5 Which of the following factors was/were responsible for impoverishment of Indian agriculture in
British times?
1. Land revenue system
2. Economic policies of British
3. Ruin of handicrafts industries

Select the correct answer using the code given below


A. 1 and 2 only
B. 3 only
C. 2 and 3 only
D. 1, 2 and 3

Correct Answer : D

Answer Justification :

All the factors were responsible.

The impoverishment of the Indian peasantry was a direct result of the transformation of the
agrarian structure due to—

● colonial economic policies,

● ruin of the handicrafts leading to overcrowding of land,

● the new land revenue system,

● colonial administrative and judicial system

6 Arrange the following peasant movements in chronological order


1. Tebhaga Movement
2. Deccan Riots
3. Indigo Revolts
4. Eka Movement

Select the correct answer using the code given below


A. 2 – 3 – 1- 4
B. 4 – 2 – 3 – 1

3
Total Marks : 200
Online Prelims TEST - 16 (SUBJECT WISE)
( InsightsIAS Mock Test Series for UPSC Preliminary Exam 2020 ) Mark Scored : 0

C. 3 – 2 – 4 – 1
D. 2 – 3 -1- 4

Correct Answer : C

Answer Justification :

Indigo Revolt (1859-60)

In Bengal, the indigo planters, nearly all Europeans, exploited the local peasants by forcing them to
grow indigo on their lands instead of the more paying crops like rice.

The ryots of Deccan region (1875) of western India suffered heavy taxation under the
Ryotwari system. Here again the peasants found themselves trapped in a vicious network with the
moneylender as the exploiter and the main beneficiary. These moneylenders were mostly
outsiders—Marwaris or Gujaratis.

The conditions had worsened due to a crash in cotton prices after the end of the American Civil War
in 1864, the Government’s decision to raise the land revenue by 50% in 1867, and a succession of
bad harvests.

Eka Movement

Towards the end of 1921, peasant discontent resurfaced in some northern districts of the United
Provinces—Hardoi, Bahraich, Sitapur.

Tebhaga Movement

In September 1946, the Bengal Provincial Kisan Sabha gave a call to implement, through mass
struggle, the Flood Commission recommendations of tebhaga—two-thirds’ share—to the bargardars,
the share-croppers also known as bagchasi or adhyar, instead of the one-half share.

7 Consider the following statements regarding Mappila Revolt


1. The Mappilas were the Muslim tenants inhabiting the Malabar region.
2. The Mappila movement merged with the Khilafat agitation.

Which of the statements given above is/are correct?


A. 1 only
B. 2 only
C. Both 1 and 2
D. Neither 1 nor 2

Correct Answer : C

4
Total Marks : 200
Online Prelims TEST - 16 (SUBJECT WISE)
( InsightsIAS Mock Test Series for UPSC Preliminary Exam 2020 ) Mark Scored : 0

Answer Justification :

Both the statements are correct.

The Mappilas were the Muslim tenants inhabiting the Malabar region where most of the
landlords were Hindus. The Mappilas had expressed their resentment against the oppression of
the landlords during the nineteenth century also.

The Mappila tenants were particularly encouraged by the demand of the local Congress body for a
government legislation regulating tenant-landlord relations. Soon, the Mappila movement
merged with the ongoing Khilafat agitation. The leaders of the Khilafat-Non-Cooperation
Movement like Gandhi, Shaukat Ali and Maulana Azad addressed Mappila meetings.

8 Consider the following statements


1. The Calcutta Madrasah was established by Jonathan Duncan
2. The Sanskrit College was established by Warren Hastings
3. Fort William College was set up by Wellesley in 1800 for training of civil servants

Which of the statements given above is/are correct?


A. 2 only
B. 3 only
C. 1, 2 and 3
D. None

Correct Answer : B

Answer Justification :

The Calcutta Madrasah was established by Warren Hastings in 1781 for the study of Muslim
law and related subjects. Hence Statement 1 is incorrect.

The Sanskrit College was established by Jonathan Duncan, the resident, at Benaras in 1791
for study of Hindu law and philosophy. Hence Statement 2 is incorrect.

Fort William College was set up by Wellesley in 1800 for training of civil servants of the
Company in languages and customs of Indians (closed in 1802).

9 Consider the following statements


1. Charter Act 1813 directed the company to sanction one lakh rupees annually for promoting
education
2. Wood’s Despatch recommended English as the medium of instruction for higher studies and
vernaculars at school level.

Which of the statements given above is/are correct?


A. 1 only
B. 2 only

5
Total Marks : 200
Online Prelims TEST - 16 (SUBJECT WISE)
( InsightsIAS Mock Test Series for UPSC Preliminary Exam 2020 ) Mark Scored : 0

C. Both 1 and 2
D. Neither 1 nor 2

Correct Answer : C

Answer Justification :

Both the statements are correct.

The Charter Act of 1813 incorporated the principle of encouraging learned Indians and
promoting knowledge of modern sciences in the country. The Act directed the Company to
sanction one lakh rupees annually for this purpose.

In 1854, Charles Wood prepared a despatch on an educational system for India. Considered
the “Magna Carta of English Education in India”, this document was the first comprehensive plan
for the spread of education in India.

It recommended English as the medium of instruction for higher studies and vernaculars at school
level

10 Which of the following was/were the recommendations of Hunter Commission?


1. It recommended transfer of control of primary education to newly set up district and municipal
boards
2. It recommended a system of grants-in-aid to encourage private enterprise.
3. It emphasised that state’s special care is required for extension and improvement of primary
education.

Which of the statements given above is/are correct?


A. 1 and 2 only
B. 1, 2 and 3
C. 3 only
D. 1 and 3 only

Correct Answer : D

Answer Justification :

The commission—

(i) emphasised that state’s special care is required for extension and improvement of
primary education, and that primary education should be imparted through vernacular.

(ii) recommended transfer of control of primary education to newly set up district and
municipal boards.

(iii) recommended that secondary (High School) education should have two divisions—

6
Total Marks : 200
Online Prelims TEST - 16 (SUBJECT WISE)
( InsightsIAS Mock Test Series for UPSC Preliminary Exam 2020 ) Mark Scored : 0

● literary—leading up to university.

● vocational—for commercial careers

Recommending a system of grants-in-aid to encourage private enterprise was an idea of


Wood’s Despatch. Hence Statement 2 is incorrect.

11 Consider the following statements


1. Raleigh Commission was set up to go into conditions and prospects of universities in India.
2. Saddler University Commission was set up to study and report on problems of Calcutta University.

Which of the statements given above is/are correct?


A. 1 only
B. 2 only
C. Both 1 and 2
D. Neither 1 nor 2

Correct Answer : C

Answer Justification :

Both the statements are correct.

In 1902, Raleigh Commission was set up to go into conditions and prospects of


universities in India and to suggest measures for improvement in their constitution and working.

Saddler University Commission was set up to study and report on problems of Calcutta
University but its recommendations were applicable more or less to other universities also.

12 Which of the following was/were the features of Wardha Scheme of Education


1. First seven years of schooling to be an integral part of a free and compulsory nationwide education
system.
2. Teaching to be in Hindi from class II to VII and in English only after class VIII.
3. Inclusion of a basic handicraft in the syllabus

Select the correct answer using the code given below


A. 1 and 2 only
B. 2 only
C. 3 only
D. 1, 2 and 3

Correct Answer : D

Answer Justification :

7
Total Marks : 200
Online Prelims TEST - 16 (SUBJECT WISE)
( InsightsIAS Mock Test Series for UPSC Preliminary Exam 2020 ) Mark Scored : 0

All the statements given above are correct.

The Congress had organised a National Conference on Education in October 1937 in Wardha. In the
light of the resolutions passed there, Zakir Hussain committee formulated a detailed national
scheme for basic education.

The scheme had the following provisions.

(i) Inclusion of a basic handicraft in the syllabus.

(ii) First seven years of schooling to be an integral part of a free and compulsory
nationwide education system (through mother tongue).

(iii) Teaching to be in Hindi from class II to VII and in English only after class VIII.

(iv) Ways to be devised to establish contact with the community around schools through service.

(v) A suitable technique to be devised with a view to implementing the main idea of basic
education—educating the child through the medium of productive activity of a suitable handicraft.

13 Which of the following reasons prompted British to promote English Education in India?
1. Need to ensure a cheap supply of educated Indians to man an increasing number of subordinate
posts in administration.
2. Educated Indians would help expand market for British manufactures in India.
3. Western education would reconcile Indians to British rule.

Select the correct answer using the code given below


A. 2 and 3 only
B. 1, 2 and 3
C. 3 only
D. 1 and 3 only

Correct Answer : B

Answer Justification :

All the statements given above are correct.

The government measures for promotion of education were influenced by—

● agitation in favour of modern education by enlightened Indians, Christian missionaries and


humanitarian officials;

● the need to ensure a cheap supply of educated Indians to man an increasing number of
subordinate posts in administration and in British business concerns—thus there was an emphasis
on English medium as the language of administration and of education;

● the hope that educated Indians would help expand market for British manufactures in India;

8
Total Marks : 200
Online Prelims TEST - 16 (SUBJECT WISE)
( InsightsIAS Mock Test Series for UPSC Preliminary Exam 2020 ) Mark Scored : 0

● an expectation that Western education would reconcile Indians to British rule, particularly as it
glorified British conquerors and their administration.

14 Consider the following statements regarding Vernacular Press Act, 1878


1. The magistrate’s action was final and no appeal could be made in a court of law.
2. Surendranath Banerjea became the first Indian journalist to be imprisoned after enactment of Act.
3. Lord Curzon repealed the act

Which of the statements given above is/are correct?


A. 1 and 2 only
B. 2 only
C. 1 and 3 only
D. 2 and 3 only

Correct Answer : A

Answer Justification :

The magistrate’s action was final and no appeal could be made in a court of law. Hence Statement
1 is correct.

The magistrate’s action was final and no appeal could be made in a court of law. A vernacular
newspaper could get exemption from the operation of the Act by submitting proofs to a government
censor.

In 1883, Surendranath Banerjea became the first Indian journalist to be imprisoned. Hence
Statement 2 is correct.

Lord Rippon repealed the act in 1882. Hence Statement 3 is incorrect.

15 Consider the following statements


1. James Augustus Hickey in 1780 started The Bengal Gazette.
2. Press Act, 1835 required a printer/publisher to give a precise account of premises of a publication
and cease functioning.

Which of the statements given above is/are correct?


A. 1 only
B. 2 only
C. Both 1 and 2
D. Neither 1 nor 2

Correct Answer : C

Answer Justification :

9
Total Marks : 200
Online Prelims TEST - 16 (SUBJECT WISE)
( InsightsIAS Mock Test Series for UPSC Preliminary Exam 2020 ) Mark Scored : 0

Both the statements are correct.

James Augustus Hickey in 1780 started The Bengal Gazette or Calcutta General
Advertiser, the first newspaper in India, which was seized in 1872 because of its outspoken
criticism of the Government.

The new Press Act (1835) required a printer/publisher to give a precise account of
premises of a publication and cease functioning, if required by a similar declaration.

16 Consider the following pairs


Newspaper Started By
The Hindu G. Subramaniya Aiyar
Amrita Bazar Patrika N.N. Sen
Sudharak Surendranath Banerjea

Which of the pairs given above is/are matched correctly?


A. 2 and 3 only
B. 1 only
C. 1 and 3 only
D. 3 only

Correct Answer : B

Answer Justification :

Many newspapers emerged during these years under distinguished and fearless journalists. These
included The Hindu and Swadesamitran under G. Subramaniya Aiyar, The Bengalee under
Surendranath Banerjea, Voice of India under Dadabhai Naoroji, Amrita Bazar Patrika under Sisir
Kumar Ghosh and Motilal Ghosh, Indian Mirror under N.N. Sen, Kesari (in Marathi) and
Maharatta (in English) under Balgangadhar Tilak, Sudharak under Gopal Krishna Gokhale, and
Hindustan and Advocate under G.P. Verma.

17 Which of the following personalities was/were the early critics of British Economic policies?
1. Dadabhai Naoroji
2. Romesh Chandra Dutt
3. Prithwishchandra Ray

Select the correct answer using the code given below


A. 1 and 3 only
B. 2 and 3 only
C. 1 and 2 only
D. 1, 2 and 3

10
Total Marks : 200
Online Prelims TEST - 16 (SUBJECT WISE)
( InsightsIAS Mock Test Series for UPSC Preliminary Exam 2020 ) Mark Scored : 0

Correct Answer : D

Answer Justification :

All the above personalities were the economic critics.

Dadabhai Naoroji, the ‘Grand Old Man of India’, who after a brilliant analysis of the colonial
economy put forward the theory of economic drain in Poverty and UnBritish Rule in India. Other
economic analysts included Justice Mahadeo Govind Ranade, Romesh Chandra Dutt (The Economic
History of India), Gopal Krishna Gokhale, G. Subramaniya Iyer and Prithwishchandra Ray.

18 Consider the following statements


1. British looked for alliances with the most reactionary of social groups like the princes, zamindars.
2. British encouraged caste and communal consciousness among the people.

Which of the statements given above is/are not correct?


A. 1 only
B. 2 only
C. Both 1 and 2
D. Neither 1 nor 2

Correct Answer : D

Answer Justification :

Both the statements are correct.

In their pursuit of reactionary policies and hope to expand their social base, the British
looked for alliances with the most reactionary of social groups—the princes, zamindars, etc.

Having decided to side with the reactionary elements of Indian society, the British
withdrew support to social reforms, which they felt had aroused the wrath of orthodox sections
against them. Also, by encouraging caste and communal consciousness, the British helped the
reactionary forces

19 Consider the following statements


1. The Indian Factory Act, 1881 provided weekly holiday for all
2. Indian Factory Act, 1891 dealt primarily with the problem of child labour

Which of the statements given above is/are correct?


A. 1 only
B. 2 only
C. Both 1 and 2
D. Neither 1 nor 2

11
Total Marks : 200
Online Prelims TEST - 16 (SUBJECT WISE)
( InsightsIAS Mock Test Series for UPSC Preliminary Exam 2020 ) Mark Scored : 0

Correct Answer : D

Answer Justification :

Both the statements are incorrect.

The Indian Factory Act, 1881 dealt primarily with the problem of child labour (between 7 and
12 years of age).

Its significant provisions were:

● employment of children under 7 years of age prohibited,

● working hours restricted to 9 hours per day for children,

● children to get four holidays in a month,

● hazardous machinery to be properly fenced off

The Indian Factory Act, 1891

● increased the minimum age (from 7 to 9 years) and the maximum (from 12 to 14 years) for
children,

● reduced maximum working hours for children to 7 hours a day,

● fixed maximum working hours for women at 11 hours per day with an one-and-a-half hour interval
(working hours for men were left unregulated),

● provided weekly holiday for all.

20 Consider the following statements


1. Warren Hastings was the first to bring into existence and organise the civil services.
2. Lytton introduced the statutory civil service.
3. In 1803, Satyendra Nath Tagore became the first Indian to qualify for the Indian Civil Service.

Which of the statements given above is/are correct?


A. 1 and 2 only
B. 2 only
C. 3 only
D. 1 and 3 only

Correct Answer : B

Answer Justification :

Cornwallis (governor-general, 1786-93) was the first to bring into existence and organise the
civil services. Hence Statement 1 is incorrect.

12
Total Marks : 200
Online Prelims TEST - 16 (SUBJECT WISE)
( InsightsIAS Mock Test Series for UPSC Preliminary Exam 2020 ) Mark Scored : 0

In 1878-79, Lytton introduced the Statutory Civil Service consisting of one-sixth of


covenanted posts to be filled by Indians of high families through nominations by local governments
subject to approval by the secretary of State and the viceroy. Hence Statement 2 is correct.

In 1863, Satyendra Nath Tagore became the first Indian to qualify for the Indian Civil Service.
Hence Statement 3 is incorrect.

21 Aitchison Committee constituted during the British time is related to

A. Judicial Reforms
B. Public Service Reforms
C. Police reforms
D. None of the above

Correct Answer : B

Answer Justification :

The Aitchison Committee on Public Services (1886), set up by Dufferin, recommended—

● dropping of the terms ‘covenanted’ and ‘uncovenanted’;

● classification of the civil service into Imperial Indian Civil Service (examination in England),
Provincial Civil Service (examination in India) and Subordinate Civil Service (examination in India);
and,

● raising the age limit to 23.

22 Consider the following statements


1. Mayo appointed an SP for each division helped by a number of spies
2. Bentinck abolished the office of the SP

Which of the statements given above is/are correct?


A. 1 only
B. 2 only
C. Both 1 and 2
D. Neither 1 nor 2

Correct Answer : C

Answer Justification :

Both the statements are correct.

1808 Mayo appointed an SP for each division helped by a number of spies (goyendas) but these

13
Total Marks : 200
Online Prelims TEST - 16 (SUBJECT WISE)
( InsightsIAS Mock Test Series for UPSC Preliminary Exam 2020 ) Mark Scored : 0

spies committed depredations on local people.

Bentinck (governor-general, 1828-35) abolished the office of the SP. The


collector/magistrate was now to head the police force in his jurisdiction and the commissioner in
each division was to act as the SP

23 Consider the following statements regarding the development of judiciary system in India
1. The Supreme Court and the Sadar Adalats were merged into three High Courts at Calcutta, Bombay
and Madras in 1865.
2. The Government of India Act, 1935 provided for a Federal Court which could settle disputes
between governments.

Which of the statements given above is/are correct?


A. 1 only
B. 2 only
C. Both 1 and 2
D. Neither 1 nor 2

Correct Answer : C

Answer Justification :

Both the statements are correct.

Developments in Judiciary.

1860: It was provided that the Europeans can claim no special privileges except in criminal cases,
and no judge of an Indian origin could try them.

1865: The Supreme Court and the Sadar Adalats were merged into three High Courts at Calcutta,
Bombay and Madras.

1935: The Government of India Act provided for a Federal Court (set up in 1937) which could settle
disputes between governments and could hear limited appeals from the High Courts.

24 Consider the following statements regarding Government of India Act, 1919


1. The Indian Legislative Council at the Centre was replaced by a bicameral system consisting of a
Council of State.
2. The Act separated for the first time the provincial and central budgets
3. Dyarchy in the provinces was abolished and provinces were given autonomy

Which of the statements given above is/are correct?


A. 1 and 2 only
B. 2 only
C. 3 only
D. 1, 2 and 3

14
Total Marks : 200
Online Prelims TEST - 16 (SUBJECT WISE)
( InsightsIAS Mock Test Series for UPSC Preliminary Exam 2020 ) Mark Scored : 0

Correct Answer : A

Answer Justification :

Under the 1919 Act, the Indian Legislative Council at the Centre was replaced by a bicameral
system consisting of a Council of State (Upper House) and a Legislative Assembly (Lower House).

The Act separated for the first time the provincial and central budgets, with provincial legislatures
being authorised to make their budgets.

Dyarchy in the provinces was abolished and provinces were given autonomy in Government
of India Act, 1935. Hence Statement 3 is incorrect.

25 Consider the following statements.


1. In the first Lok Sabha, the single largest party in the opposition was the Swatantra Party.
2. The Bharatiya Jana Sangh was formed in 1951 with Deen Dayal Upadhyaya as its founder-President.
3. Congress won a majority of seats in all the states including Travancore-Cochin, Madras and Orissa
in first general elections

Select the correct answer using the code given below


A. 2 only
B. 2 and 3 only
C. 1 only
D. None

Correct Answer : D

Answer Justification :

Congress party won 364 of the 489 seats in the first Lok Sabha and finished way ahead of any other
challenger. The Communist Party of India that came next in terms of seats won only 16
seats. Hence Statement 1 is incorrect.

The state elections were held with the Lok Sabha elections. The Congress scored big victory in
those elections as well. It won a majority of seats in all the states except Travancore-Cochin (part of
today’s Kerala), Madras and Orissa. Finally even in these states the Congress formed the
government. Hence statement 3 is incorrect.

The Bharatiya Jana Sangh was formed in 1951 with Shyama Prasad Mukherjee as its founder-
President. Hence Statement 2 is incorrect.

26 Attlee’s statement got lot of importance in India’s Freedom Struggle. What is the significance of it?

A. It declared the British intention of leaving the Indian subcontinent.


B. It transferred the power completely to the princely states.

15
Total Marks : 200
Online Prelims TEST - 16 (SUBJECT WISE)
( InsightsIAS Mock Test Series for UPSC Preliminary Exam 2020 ) Mark Scored : 0

C. Release of political prisoners of Freedom struggle


D. None of the above

Correct Answer : A

Answer Justification :

Clement Attlee, the British prime minister, sensing the trouble all around, made an announcement
on February 20, 1947. The British House of Commons declared the British intention of
leaving the Indian subcontinent.

A deadline of June 30, 1948 was fixed for transfer of power even if the Indian politicians had not
agreed by that time on the constitution.

27 Consider the following statements regarding Mountbatten Plan


1. Independence for princely states was granted in the Plan.
2. Accession of Hyderabad to Pakistan was ruled out in the Plan.
3. A boundary commission to be set up if partition was to be effected.

Which of the statements given above is/are correct?


A. 2 only
B. 1 and 3 only
C. 3 only
D. 2 and 3 only

Correct Answer : D

Answer Justification :

Mountbatten Plan

(i) independence for princely states ruled out—they would join either India or Pakistan;
Hence Statement 1 is incorrect.

(ii) independence for Bengal ruled out;

(iii) accession of Hyderabad to Pakistan ruled out

(Mountbatten supported the Congress on this);

(iv) freedom to come on August 15, 1947; and

(v) a boundary commission to be set up if partition was to be effected.

28 Which of the following personalities defended INA prisoners in the court of law
1. Dr. Ambedkar

16
Total Marks : 200
Online Prelims TEST - 16 (SUBJECT WISE)
( InsightsIAS Mock Test Series for UPSC Preliminary Exam 2020 ) Mark Scored : 0

2. Jawharlal Nehru
3. Asaf Ali

Select the correct answer using the code given below


A. 1 and 3 only
B. 2 only
C. 3 only
D. 2 and 3 only

Correct Answer : D

Answer Justification :

At the first post-War Congress session in September 1945 at Bombay, a strong resolution
was adopted declaring Congress support for the INA cause.

Defence of INA prisoners in the court was organised by Bhulabhai Desai, Tej Bahadur Sapru,
Kailash Nath Katju, Jawaharlal Nehru and Asaf Ali.

29 Which of the following was/were recommendations of Cabinet Mission Plan?


1. Rejection of the demand for a full-fledged Pakistan.
2. Provinces were to have full autonomy and residual powers.
3. Communal questions in the central legislature were to be decided by a simple majority

Select the correct answer using the code given below


A. 1 and 2 only
B. 2 only
C. 3 only
D. 1, 2 and 3

Correct Answer : D

Answer Justification :

All the statements given above are correct.

Cabinet Commission Recommendations:

Rejection of the demand for a full-fledged Pakistan

Communal questions in the central legislature were to be decided by a simple majority of both
communities present and voting.

Provinces were to have full autonomy and residual powers.

17
Total Marks : 200
Online Prelims TEST - 16 (SUBJECT WISE)
( InsightsIAS Mock Test Series for UPSC Preliminary Exam 2020 ) Mark Scored : 0

30 “Direct Action” call by Muslim League aimed at

A. Achieving Independence from British before August 15th, 1947.


B. To secure the interests of Muslim by ensuring more representation in the Parliament.
C. To achieve state of Pakistan out of India.
D. None of the above

Correct Answer : C

Answer Justification :

The League withdrew its acceptance of the long-term plan in response to Nehru’s statement and
gave a call for “direct action” from August 16 to achieve Pakistan.

31 Consider the following statements regarding ‘Potti Sriramulu’


1. He was a social activist worked for emancipation of dalits
2. Participated in individual satyagraha including salt satyagraha
3. His struggle led to the formation of Andhra Pradesh with its capital Kurnool

Select the correct answer using the code given below


A. 1 only
B. 1 and 3 only
C. 1, 2 and 3
D. 1 and 2 only

Correct Answer : C

Answer Justification :

Potti Sriramulu (1901-1952): Gandhian worker, he left the government job to participate in Salt
Satyagraha. Sreeramulu took part in the Indian Independence Movement and was
imprisoned for participating in the 1930 Salt Satyagraha.

He undertook three fasts, during 1946-1948, in support of Dalit (an oppressed Hindu caste
then referred to by Gandhi and his supporters by the contentious, though well-intentioned,
term Harijan) rights to enter holy places, such as the temples of Nellore. He fasted in support of
Dalit entry rights to the Venu Gopala Swamy Temple in Moolapeta, Nellore, rights which were
eventually secured

He became famous for undertaking a hunger strike in support of the formation of an Indian state for
the Telugu-speaking population of Madras Presidency; he lost his life in the process. His death
sparked public rioting and Indian Prime Minister Jawaharlal Nehru declared the intent by the newly
liberated nation to form Andhra State. On 1 October 1953, the Telugu speaking Andhra
State was established with its capital in Kurnool, three days following the death of
Sreeramulu.

18
Total Marks : 200
Online Prelims TEST - 16 (SUBJECT WISE)
( InsightsIAS Mock Test Series for UPSC Preliminary Exam 2020 ) Mark Scored : 0

Hence all the statements are correct.

32 Consider the following events


1. Operation Meghadoot
2. Operation Pawan
3. Operation Cactus
4. Operation Polo

Which of the following is the correct chronological sequence of the above events?
A. 4 — 2 — 1 — 3
B. 3 — 2 — 1 — 4
C. 4 — 2 — 1 — 3
D. 4 — 1 — 2 — 3

Correct Answer : D

Answer Justification :

Major Military Missions after 1950 (Republic of India)

Operation Polo

It is the code name of the Hyderabad "police action" in September 1948, by the newly independent
India against the Hyderabad State. It was a military operation in which the Indian Armed Forces
invaded the Nizam-ruled princely state, annexing it into the Indian Union.

Operation Meghadoot (1984)

In the late 1970s and early 1980s, Pakistan began organising tourist expeditions to the Siachen
Glacier, disputed territory with India. Irked by this development, in April 1984 India initiated the
successful.

Operation Meghdoot during which it gained control over all of the 70 kilometer (41 mile)-long
Siachen Glacier, and all of its tributary glaciers, as well as the three main passes of the Saltoro
Ridge immediately west of the glacier—Sia La, Bilafond La, and Gyong La. It has been claimed that,
India gained more than 1,000 square miles (3,000 km2) of territory as a result of its military
operations in Siachen.

In 1987 and in 1989 Pakistan to re-take the glacier but was unsuccessful. The conflict ended with
Indian Victory. There has been a ceasefire since 2003.

Operation Pawan (1987-1990)

The Indian Peace Keeping Force (IPKF) carried out a mission in northern and eastern Sri Lanka in
1987–1990 to disarm the Tamil Tigers under the terms of the Indo-Sri Lanka Accord. It was a
difficult battle for the Indian Army, which was not trained for an unconventional war.

After losing approximately 1,200 personnel and several T-72 tanks, India ultimately abandoned the

19
Total Marks : 200
Online Prelims TEST - 16 (SUBJECT WISE)
( InsightsIAS Mock Test Series for UPSC Preliminary Exam 2020 ) Mark Scored : 0

mission in consultation with the Sri Lankan government. In what was labelled as Operation Pawan,
the Indian Air Force flew about 70,000 sorties to and within Sri Lanka.

Operation Cactus (1998)

In 1988 a group of Maldivians led by Abdullah Luthufi tried to overthrow then Maldivian
Government. The attempt was a failure due to the intervention of the Indian Armed Forces. The
operation was code named as Operation Cactus. INS Godavari and Betwa were used by the Indian
Navy in the operation

33 Consider the following statements regarding Government of India Act, 1935


1. There was a provision for joint sitting in cases of deadlock between the houses
2. Provinces derived their power and authority directly from the British Crown
3. The India Council of the Secretary of State was abolished

Which of the statements given above is/are correct?


A. 1 and 2 only
B. 2 only
C. 3 only
D. 1, 2 and 3

Correct Answer : D

Answer Justification :

All the statements given above are correct.

There was a provision for joint sitting in cases of deadlock between the houses. There were to be
three subjectlists—the Federal Legislative List, the Provincial Legislative List and the Concurrent
Legislative List. Residuary legislative powers were subject to the discretion of the governor-general.

Provinces derived their power and authority directly from the British Crown. They were given
independent financial powers and resources. Provincial governments could borrow money on their
own security.

The India Council of the Secretary of State was abolished.

34 Consider the following statements:


1. The Independent Labour Party (ILP) was a political organisation formed under the leadership of B.
R. Ambedkar.
2. 2nd general elections was the best performance of Communist party of India so far in terms of Loka
Sabha seats.

Which of the statements given above is/are correct?


A. 1 only
B. 2 only

20
Total Marks : 200
Online Prelims TEST - 16 (SUBJECT WISE)
( InsightsIAS Mock Test Series for UPSC Preliminary Exam 2020 ) Mark Scored : 0

C. Both 1 and 2
D. Neither 1 nor 2

Correct Answer : A

Answer Justification :

The Independent Labour Party (ILP) was a political organisation formed under the
leadership of B. R. Ambedkar on 15 August 1936. It opposed the brahmanical and capitalist
structures in India, supported the Indian working class and sought to dismantle the caste system.
Hence option (a) is correct

35 Consider the following events regarding formation of states in India.


1. Nagaland separated out of Assam as an independent state
2. Gujarat separated out of Bombay as an independent state.
3. Meghalaya converted to state from sub state in Assam
4. Sikkim from being a protectorate of India became a full state

Which of the following is the correct chronological sequence of the above events?
A. 2 — 3 — 1 — 4
B. 3 — 2 — 1 — 4
C. 4 — 2 — 1 — 3
D. 2 — 1 — 3— 4

21
Total Marks : 200
Online Prelims TEST - 16 (SUBJECT WISE)
( InsightsIAS Mock Test Series for UPSC Preliminary Exam 2020 ) Mark Scored : 0

Correct Answer : D

Answer Justification :

1953: Andhra Pradesh carved out of Madras

1960: Gujarat separated out of Bombay(15th State)

1963: Nagaland separated out of Assam(16th State)

1966: Haryana separated out of Punjab(17th State) HP separated as UT

1971: HP converted into state from UT(18th state)

1972: Manipur converted to state from UT(19th)

1972: Tripura converted to state from UT(20th)

1972: Meghalaya converted to state from sub state in Assam(21st)

1975: Sikkim from being a protectorate of India became a full state(22nd)

1987: Mizoram converted to state from UT(23rd)

1987: Arunanchal converted to state from UT(24th)

1987: Goa converted to state from UT(25th)

2000: Chhatisgarh (26th)

2000: Uttarakhand (27th)

2000: Jharkhand (28th)

36 Which of the following events connected declaration of National Emergency 1975?


1. Emergence of Congress syndicate
2. Railways strike by George Fernandez
3. Keshavananda Bharati Judgement

Select the correct answer using the code given below


A. 1, 2 and 3
B. 1 and 2 only
C. 2 and 3 only
D. 2 only

Correct Answer : C

22
Total Marks : 200
Online Prelims TEST - 16 (SUBJECT WISE)
( InsightsIAS Mock Test Series for UPSC Preliminary Exam 2020 ) Mark Scored : 0

Answer Justification :

The Indian National Congress (Organisation) or Congress (O) or Congress Syndicate was a political
party in India formed when the Congress party split following the expulsion of Indira Gandhi.

On 12 November 1969, the Prime Minister of India Indira Gandhi was expelled from the Congress
party for violating the party discipline. The party finally split with Indira Gandhi setting up a rival
organization, which came to be known as Congress (R).

Establishment of congress syndicate is not connected with declaration of emergency.


Hence Statement 1 is incorrect.

37 Consider the following statements regarding Bhoodan Movement.


1. It was a voluntary land reform movement, started by Acharya Vinoba Bhave
2. Bhave received first land donation at Pochampalli village in Telangana Region
3. Jayaprakash Narayan give up active politics and joined Bhoodan movement in 1953.

Which of the statements given above is/are correct?


A. 1 only
B. 1 and 3 only
C. 2 and 3 only
D. 1, 2 and 3

Correct Answer : D

Answer Justification :

All the statements are correct.

The Bhoodan Movement or Land Gift Movement, was a voluntary land reform movement in
India, started by Acharya Vinoba Bhave in 1951 at Pochampally village which is now in Telangana,
India and known as Bhoodan Pochampally.

38 Who headed the Aarzi Hukumat of the people of Junagadh during the annexation of Junagadh
princely state into India?

A. Samaldas Gandhi
B. Vallabhbhai Patel
C. Moraji Desai
D. Nawab Sahab of Junagadh.

Correct Answer : A

Answer Justification :

23
Total Marks : 200
Online Prelims TEST - 16 (SUBJECT WISE)
( InsightsIAS Mock Test Series for UPSC Preliminary Exam 2020 ) Mark Scored : 0

Junagadh was a princely state of British India, located in what is now Gujarat, outside but under
the suzerainty of British India.

In the independence and partition of British India of 1947, the 552 princely states were given a
choice to either join the new Dominion of India or the newly formed state of Pakistan.

The Nawab of Junagadh, Muhammad Mahabat Khanji III, a Muslim whose ancestors had ruled
Junagadh and small principalities for some two hundred years, decided that Junagadh should
become part of Pakistan, much to the displeasure of many of the people of the state, an
overwhelming majority of whom were Hindus. Vallabhbhai Patel offered Pakistan time to reverse its
acceptance of the accession and to hold a plebiscite in Junagadh.

Samaldas Gandhi formed a government-in-exile, the Aarzi Hukumat ) of the people of


Junagadh. Eventually, Patel ordered the forcible annexation of Junagadh's three
principalities. Junagadh's state government, facing financial collapse and lacking forces with
which to resist Indian force, invited the Government of India to take control. A plebiscite was
conducted in December, in which approximately 99.95% of the people chose India over Pakistan.

39 Consider the following events


1. Agra Summit
2. Operation Shakti
3. Attack on Indian Parliament
4. United States Invasion of Afghanistan

Which of the following is the correct chronological sequence of the above events?
A. 2 — 4 — 1 — 3
B. 2 — 3 — 1 — 4
C. 1— 2 — 3 — 4
D. 2 — 1 — 4 — 3

Correct Answer : D

Answer Justification :

On May 11, 1998 and May 13, 1998, India successfully test fired the Shakti-I, II, III, IV, and V
nuclear missiles. The operation was led by aerospace engineer and late president DR ABJ Abdul
Kalam.

The Agra summit was a historic two-day summit meeting between India and Pakistan which lasted
from 14–16 July 2001. It was organized with the aim of resolving long-standing issues between India
and Pakistan.

The United States invasion of Afghanistan occurred after the September 11 attacks in late 2001,
supported by close US allies. The conflict is also known as the U.S. war in Afghanistan. Its public
aims were to dismantle al-Qaeda, and to deny it a safe base of operations in Afghanistan by
removing the Taliban from power.

24
Total Marks : 200
Online Prelims TEST - 16 (SUBJECT WISE)
( InsightsIAS Mock Test Series for UPSC Preliminary Exam 2020 ) Mark Scored : 0

The 2001 Indian Parliament attack was a terrorist attack at the Parliament of India in New Delhi on
13 December 2001. The perpetrators were Lashkar-e-Taiba and Jaish-e-Mohammed, two terrorist
organizations were involved.

40 Which of the following statements regarding “Non- Alignment Movement” (NAM) is/are correct?
1. NAM philosophy is based on the passive political response and neutrality than active involvement in
world politics.
2. Iran had hosted the last NAM summit in 2019.
3. Dr. Ambedkar was critical of NAM policy and advocated for pro-US foreign policy.

Select the correct answer using the code given below


A. 3 only
B. 1 only
C. 1 and 3 only
D. 2 and 3 only

Correct Answer : A

Answer Justification :

The Non-Aligned Movement (NAM) is a group of states that are not formally aligned with or
against any major power bloc. As of 2012, the movement has 120 members.

It was established in 1961 in Belgrade, Yugoslavia. An initiative of Yugoslav president Josip Broz
Tito and Indian Prime Minister Jawaharlal Nehru led to the first Conference of Heads of State or
Government of Non-Aligned Countries

NAM objective is to actively participate in the world politics, but not aligning with any of
the power bloc. Hence statement 1 is incorrect.

Last Summit

Azerbaijan had hosted the last NAM summit in 2019.

There were some parties and groups in the country that believed that India should be friendlier with
the bloc led by the US because that bloc claimed to be pro-democracy. Among those who thought
on these lines were leaders like Dr Ambedkar.

41 Consider the following statements regarding Quit India Movement


1. The Quit India Resolution was ratified at the Congress meeting at Gowalia Tank, Calcutta.
2. It resolved to demand an immediate end to British rule in India.
3. Jawaharlal Nehru was named as the leader of the movement

Which of the statements given above is/are correct?


A. 1 and 2 only
B. 2 only

25
Total Marks : 200
Online Prelims TEST - 16 (SUBJECT WISE)
( InsightsIAS Mock Test Series for UPSC Preliminary Exam 2020 ) Mark Scored : 0

C. 3 only
D. 2 and 3 only

Correct Answer : B

Answer Justification :

The Quit India Resolution was ratified at the Congress meeting at Gowalia Tank, Bombay, on
August 8, 1942. Hence Statement 1 is incorrect.

The meeting also resolved to

● demand an immediate end to British rule in India.

● declare commitment of free India to defend itself against all types of Fascism and imperialism.

● form a provisional Government of India after British withdrawal.

● sanction a civil disobedience movement against British rule.

Gandhi was named the leader of the struggle. Hence Statement 3 is incorrect.

42 Consider the following statements


1. Rammanohar Lohia, Jayaprakash Narayan were largely involved in underground activities during
Quit India Movement.
2. Parallel governments were established in many places like Satara, Midnapore.

Which of the statements given above is/are correct?


A. 1 only
B. 2 only
C. Both 1 and 2
D. Neither 1 nor 2

Correct Answer : C

Answer Justification :

Both the statements are correct.

The main personalities taking up underground activity were Rammanohar Lohia, Jayaprakash
Narayan, Aruna Asaf Ali, Usha Mehta, Biju Patnaik, Chhotubhai Puranik, Achyut Patwardhan etc.

Parallel governments were established at many places:

● Ballia (in August 1942 for a week)—under Chittu Pandey. He got many Congress leaders
released.

26
Total Marks : 200
Online Prelims TEST - 16 (SUBJECT WISE)
( InsightsIAS Mock Test Series for UPSC Preliminary Exam 2020 ) Mark Scored : 0

● Tamluk (Midnapore, from December 1942 to September 1944)—Jatiya Sarkar undertook cyclone
relief work, sanctioned grants to schools, supplied paddy from the rich to the poor, organised Vidyut
Vahinis, etc.

● Satara (mid-1943 to 1945)—named “Prati Sarkar”, was organised under leaders like Y.B. Chavan,
Nana Patil, etc.

Village libraries and Nyayadan Mandals were organised, prohibition campaigns were carried on and
‘Gandhi marriages’ were organised

43 Consider the following statements regarding Subhash Chandra Bose


1. He passed the Indian Civil Services examination securing fourth position but resigned from the
service in 1921.
2. His political guru was Tilak.
3. Bose met Hitler under the pseudo name, Orlando Mazzotta.

Which of the statements given above is/are correct?


A. 2 and 3 only
B. 1 and 2 only
C. 3 only
D. 1 and 3 only

Correct Answer : D

Answer Justification :

Subhash Chandra Bose was an intrepid man. He had always shown a militant streak and reacted
violently to any insult of Indians by the Europeans. He passed the Indian Civil Services examination
securing fourth position but resigned from the service in 1921 to join the struggle for freedom by
becoming a member of the Congress.

His political guru was Chittaranjan Das. Hence Statement 2 is incorrect.

He became mayor of Calcutta in 1923. He was jailed many times by the British. Once it became
clear to Subhash Chandra Bose that he could not follow Gandhi’s way but that the Congress was
determined to follow Gandhi, Bose decided to go his own way to fight for independence.

Bose met Hitler under the pseudo name, Orlando Mazzotta.

44 Consider the following statements


1. Poona Pact abandoned the idea of separate electorates for the depressed classes.
2. It achieved the desired goal of emancipation of the depressed class.

Which of the statements given above is/are correct?


A. 1 only
B. 2 only

27
Total Marks : 200
Online Prelims TEST - 16 (SUBJECT WISE)
( InsightsIAS Mock Test Series for UPSC Preliminary Exam 2020 ) Mark Scored : 0

C. Both 1 and 2
D. Neither 1 nor 2

Correct Answer : A

Answer Justification :

The Poona Pact abandoned the idea of separate electorates for the depressed classes. But
the seats reserved for the depressed classes were increased from 71 to 147 in provincial
legislatures and to 18 per cent of the total in the Central Legislature.

The Poona Pact was accepted by the government as an amendment to the Communal Award.

The Poona Pact, despite giving certain political rights to the depressed classes, could not
achieve the desired goal of emancipation of the depressed class. It enabled the same old
Hindu social order to continue and gave birth too many problems. Hence Statement 2 is
incorrect.

45 Consider the following statements


1. The third round table conference was attended by the Indian National Congress and Gandhi.
2. Round table conference was the first conference arranged between the British and the Indians as
equals.

Which of the statements given above is/are correct?


A. 1 only
B. 2 only
C. Both 1 and 2
D. Neither 1 nor 2

Correct Answer : B

Answer Justification :

The first Round Table Conference was held in London between November 1930 and January 1931.
It was opened officially by King George V on November 12, 1930 and chaired by Ramsay
MacDonald.

This was the first conference arranged between the British and the Indians as equals.
Hence Statement 2 is correct.

The Congress and some prominent business leaders refused to attend, but many other groups of
Indians were represented at the conference.

The third Round Table Conference, held between November 17, 1932 and December 24,
1932, was not attended by the Indian National Congress and Gandhi. It was ignored by
most other Indian leaders. Hence Statement 1 is incorrect.

28
Total Marks : 200
Online Prelims TEST - 16 (SUBJECT WISE)
( InsightsIAS Mock Test Series for UPSC Preliminary Exam 2020 ) Mark Scored : 0

46 Which of the following factors was/were influenced the growth of Modern Nationalism.
1. French Revolution.
2. Indian Renaissance.
3. Offshoot of modernization initiated by the British in India.
4. Strong reaction to British imperialist policies in India.

Which of the statements given above is/are correct?


A. 1 and 2 only
B. 2 and 3 only
C. 1, 2 and 4 only
D. 1, 2, 3 and 4 only

Correct Answer : D

Answer Justification :

The rise and growth of Indian nationalism has been traditionally explained in terms of Indian
response to the stimulus generated by the British Raj through creation of new institutions, new
opportunities, resources, etc. In other words, Indian nationalism grew partly as a result of colonial
policies and partly as a reaction to colonial policies. In fact, it would be more correct to see Indian
nationalism as a product of a mix of various factors:

Worldwide upsurge of the concepts of nationalism and right of self-determination initiated by


the French Revolution. Hence, statement 1 is correct.

Indian Renaissance. Hence, statement 2 is correct.

Offshoot of modernisation initiated by the British in India. Hence, statement 3 is correct.

Strong reaction to British imperialist policies in India. Hence, statement 4 is correct.

47 The Indian League was started by

A. Surendranath Banerjee
B. Ananda Mohan Bose
C. Sisir Kumar Ghosh
D. G. Subramania Iyer

Correct Answer : C

Answer Justification :

29
Total Marks : 200
Online Prelims TEST - 16 (SUBJECT WISE)
( InsightsIAS Mock Test Series for UPSC Preliminary Exam 2020 ) Mark Scored : 0

The Indian League was started Ghosh with the object of “stimulating the sense of nationalism
amongst the people” and of encouraging political education. Hence, option C is correct.

48 The Seva Sadon was founded by

A. Naoroji Furdonji
B. S.S. Bangal
C. J.B. Wacha
D. Behramii M Malabari

Correct Answer : D

Answer Justification :

A Parsi Social Reformer, Beharmji M. Malabari has campaigned against evils of child marriage
and enforced widowhood all his life. For these social evils, he placed the blame on greedy priestly
class for the inaccurate interpretation of scriptures and main Hindu superstition of considering a
girl to be a serpent in the house after age of 10.

Therefore, he founded Seva Sadan in 1885 with an aim to look after socially discarded and exploited
women from all castes and providing welfare services such as education and medical treatment.
Hence, Option D is correct.

49 With reference to Swami Dayanand Saraswati, consider the following statements


1. He started the Suddhi Movement to bring back the Hindu converted to other religions.
2. He accepted both doctrine of karma and the theory of Niyati (fatalism).

Which of the statements given above is/are correct?


A. 1 only
B. 2 only
C. Both 1 and 2
D. Neither 1 nor 2

Telegram Channel
Correct Answer : A
https://t.me/visionpt3652019
Answer Justification :

Dayanand Saraswati was an Indian philosopher, social leader and founder of the Arya Samaj, a
reform movement of the Vedic dharma. He was the first to give the call for Swaraj as "India for
Indians" in 1876, a call later taken up by Lokmanya Tilak.

He started the Suddhi Movement to bring back the Hindu converted to other religions and played
some role in the growth of communalism in the 20th century. Hence, statement 1 is correct.

He accepted the doctrine of karma, but rejected the theory of Niyati (fatalism). Hence, statement

30
Total Marks : 200
Online Prelims TEST - 16 (SUBJECT WISE)
( InsightsIAS Mock Test Series for UPSC Preliminary Exam 2020 ) Mark Scored : 0

2 is incorrect.

50 Consider the following statements regarding British Indian Association


1. It was formed by the amalgamation of the Landholders’ Society and the East India Association.
2. The object of the Association was to secure improvements in the local administration of the country
and in the system of government laid down by Parliament.

Which of the statements given above is/are correct?


A. 1 only
B. 2 only
C. Both 1 and 2
D. Neither 1 nor 2

Correct Answer : C

Answer Justification :

British Indian Association was founded on October 29, 1851 at Calcutta with Raja Radhakanta Dev
and Debendranath Tagore as its President and Secretary respectively. Its membership was kept
exclusive to Indians.

It was formed by the amalgamation of the Landholders’ Society and the Bengal British India
Society. Hence, statement 1 is correct.

The object of the Association was 'to secure improvements in the local administration of the country
and in the system of government laid down by Parliament'. The Association gave leadership to
remove the existing defects in the laws and civil administration of the country and to promote
greater welfare of the Indians. Hence, statement 2 is correct.

51 With reference to Murshid Quli Khan, consider the following statements:


1. He introduced ijara system in Bengal province.
2. He resumed the jagirs and converted them into Khas lands directly under the government.

Which of the statements given above is/are correct?


A. 1 only
B. 2 only
C. Both 1 and 2
D. Neither 1 nor 2

Correct Answer : C

Answer Justification :

Both statements given above are correct.

31
Total Marks : 200
Online Prelims TEST - 16 (SUBJECT WISE)
( InsightsIAS Mock Test Series for UPSC Preliminary Exam 2020 ) Mark Scored : 0

When Murshid Quli Khan assumed the Diwani of Bengal he found that the finances of the
province were at very bad state.

To rationalize the revenue system he adopted two-fold measure

1. He introduced ijara system, i.e. contract system by which contracts of the collection of
land revenue were granted to ijaradars, i.e. contractors by taking security bonds from them.
This was called by him mal zamini system. The second or the third generation of these contractors
came to be known as zamindars.

2. He resumed the jagirs and converted them into Khas lands directly under the
government and gave the dispossessed officers fresh jagirs in the Orissa province.

52 Consider the following statements:


1. Francisco De Almedia captured Goa from the ruler of Bijapur.
2. Albuquerque initiated the ‘Blue Water’ policy.

Which of the statements given above is/are correct?


A. 1 only
B. 2 only
C. Both 1 and 2
D. Neither 1 nor 2

Correct Answer : D

Answer Justification :

Both statements given above are incorrect.

De Almedia was appointed as the first Portuguese viceroy in India in 1505 and he initiated the
‘Blue Water’ policy or the mastery of sea by strong naval power.

In 1509, De Almedia was followed by Albuquerque as the second viceroy and he was in that
power till 1515. He captured Goa from the ruler of Bijapur in 1510 and it became the
headquarters of the Portuguese in India.

53 With reference to the Carnatic Wars, consider the following statements:


1. The First Carnatic war was ended with signing of the treaty of Aix-la-Chapelle.
2. During second Carnatic war the French recalled Dupleix.

Which of the statements given above is/are correct?


A. 1 only
B. 2 only
C. Both 1 and 2
D. Neither 1 nor 2

32
Total Marks : 200
Online Prelims TEST - 16 (SUBJECT WISE)
( InsightsIAS Mock Test Series for UPSC Preliminary Exam 2020 ) Mark Scored : 0

Correct Answer : C

Answer Justification :

Both statements given above are correct.

The First Carnatic War had its background in the War of Austrian Succession (1740-48) which was
raging in Europe.

The First Carnatic war was ended with signing of the treaty of Aix-la-Chapelle. By the
terms of the treaty Madras was restored to the English. In this way the first phase of the
struggle between the English and the French on the Indian soil ended without any territorial loss or
gain to any of the Parties.

During second Carnatic war the French recalled Dupleix. Godehu was sent to supersede
Dupleix, and he arrived on August 1, 1754. Godehu at once opened negotiations with the
English and a treaty was signed stipulating that French or the English would not interfere in the
quarrels among the native princes, and both parties were left in occupation of territories actually
under their respective possession.

54 Which of the following battle ended with the Treaty of Allahabad?

A. Battle of Plassey
B. Battle of Buxar
C. First Anglo-Sikh war
D. Third Carnatic War

Correct Answer : B

Answer Justification :

The Treaty of Allahabad was signed on August 16, 1765 between Mughal Emperor Shah Alam
II of Bengal and Lord Clive of the British East India Company after the Battle of Buxar
(1764). Based on the terms of the agreement, Shah Alam II granted Diwani rights to the East India
Company. Hence, option (b) is correct.

55 With reference to Raja Sawai Jai Singh (Jai Singh II), consider the following statements:
1. He initiated socio –religious reforms such as the abolition of Sati.
2. He commissioned the Jantar Mantar observatories at multiple places in India.
3. He constructed the city of Ahmadabad.

Which of the statements given above is/are correct?


A. 1 and 2 only
B. 3 only
C. 2 only

33
Total Marks : 200
Online Prelims TEST - 16 (SUBJECT WISE)
( InsightsIAS Mock Test Series for UPSC Preliminary Exam 2020 ) Mark Scored : 0

D. 1, 2 and 3

Correct Answer : A

Answer Justification :

Jai Singh II (3 November 1688 – 21 September 1743) was he Hindu Rajput ruler of the kingdom of
Amber, he later founded the fortified city of Jaipur and made it his capital. Hence,
statement 3 is incorrect.

Ahmedabad was founded by the, ruler of Gujarat, Sultan Ahmed Shah in 1411 AD.

Being initiated in the Nimbarka Sampradaya of the Vaishnava religion, Raja Sawai Jai Singh
also promoted Sanskrit learning and initiated reforms in Hindu society like the abolition
of Sati and curbing the wasteful expenditures in Rajput weddings. Hence, statement 1 is
correct.

He built five observatories at Delhi, Mathura (in his Agra province), Benares, Ujjain (capital
of his Malwa province), and his own capital of Jaipur. His astronomical observations were
remarkably accurate.

Hence, statement 2 is correct.

56 Consider the following statements:


1. The Battle of Colachel was fought between the Indian kingdom of Travancore and the British East
India Company.
2. The Serampur Mission Press was established by the Dutch missionaries.

Which of the statements given above is/are correct?


A. 1 only
B. 2 only
C. Both 1 and 2
D. Neither 1 nor 2

Correct Answer : D

Answer Justification :

Both statements given above are incorrect.

The Battle of Colachel was fought on 10 August 1741 between the Indian kingdom of
Travancore and the Dutch East India Company, during the Travancore-Dutch War. Travancore,
under Raja Marthanda Varma, defeated the Dutch East India Company.

The Serampur Mission Press was a book and newspaper publisher that operated in
Serampur, Danish India, from 1801 to 1837. The Press was founded by William Carey, William

34
Total Marks : 200
Online Prelims TEST - 16 (SUBJECT WISE)
( InsightsIAS Mock Test Series for UPSC Preliminary Exam 2020 ) Mark Scored : 0

Ward, and other British Baptist missionaries at the Serampur Mission.

57 With reference to the British policy of Subsidiary Alliance system, consider the following statements:
1. It was given definite shape under the Governor Generalship of Lord Wallesley.
2. Under the system, the ruler of the allying Indian state was compelled to accept the permanent
stationing of a British force within his territory and to pay a subsidy for its maintenance.
3. It usually provided that the Indian ruler would agree to the posting at his court of a British
Resident.

Which of the statements given above is/are correct?


A. 2 and 3 only
B. 1 only
C. 2 only
D. 1, 2 and 3

Correct Answer : D

Answer Justification :

While the practice of helping an Indian ruler with a paid British force was quite old, it was given
definite shape by Wellesley who used it to subordinate the Indian states to the paramount
authority of the Company. Hence, statement 1 is correct.

Under his Subsidiary Alliance system, the ruler of the allying Indian state was compelled
to accept the permanent stationing of a British force within his territory and to pay a
subsidy for its maintenance. All this was done allegedly for his protection but was, in fact, a form
through which the Indian ruler paid tribute to the Company. Hence, statement 2 is correct.

The ‘Subsidiary Treaty’ usually also provided that the Indian ruler would agree to the
posting at his court of a British Resident, that he would not employ any European in his
service without the approval of the British, and that he would not negotiate with any other
Indian ruler without consulting the Governor-General. Hence, statement 3 is correct.

In return, the British undertook to defend the ruler from his enemies. They also promised non-
interference in the internal affairs of the allied state, but this was a promise they seldom kept.

58 Consider the following statements:


1. First Anglo Sikh war was fought between British and Ranjit Singh.
2. By the treaty of Amritsar Ranjit Singh promised not to expand east of the Sutlej.
3. Lord Dalhousie annexed the whole of the Punjab to the British Empire after the Second Anglo-Sikh
war.

Which of the statements given above is/are correct?


A. 2 only
B. 1 only
C. 2 and 3 only

35
Total Marks : 200
Online Prelims TEST - 16 (SUBJECT WISE)
( InsightsIAS Mock Test Series for UPSC Preliminary Exam 2020 ) Mark Scored : 0

D. 1, 2 and 3

Correct Answer : C

Answer Justification :

The loose confederation of the Sikhs of Punjab was unified into a compact powerful unit
by Ranjit Singh. He expanded his empire through conquests. To check his advance beyond the
Sutlej the East India Company persuaded Ranjit Singh to sign the Treaty of Amritsar
(1809). By this treaty he promised not to expand east of the Sutlej and confine his
conquests to the north. Hence, statement 2 is correct.

After Ranjit Singh’s death, the Punjab went through a period of chaos and confusion. In 1843,
Ranjit Singh’s minor son, Dalip Singh, became the ruler with Rani Jindan as the Regent.
To weaken the army and keep it engaged, Rani Jindan deliberately encouraged the army to
cross the river Sutlej and attack the English. In December 1845 the Sikh army crossed the
Sutlej and invaded the Company’s territories. This led to the First Anglo-Sikh War
(1845-1846). Sikh soldiers fought very bravely but they were completely defeated. The British
army occupied Lahore. War ended with signing of treaty of Lahore.

Hence, statement 1 is incorrect.

British control over the Punjab aroused a lot of resentment among the Sikhs. In 1848 a number of
revolts against the British broke out in the Punjab. The Governor-General, Lord Dalhousie,
declared war. The Sikh army which had been reduced by the Treaty of Lahore was
completely crushed. By a proclamation in 1849, Lord Dalhousie annexed the whole of the
Punjab to the British Empire. Dalip Singh was pensioned off. Hence, statement 3 is
correct.

59 With reference to Tipu Sultan, consider the following statements:


1. He commissioned a military manual called Fathul Mujahidin.
2. Tipu Sultan laid the foundation for the construction of the present-day Krishna Raja Sagara or KRS
dam on the Kaveri river.

Which of the statements given above is/are correct?


A. 1 only
B. 2 only
C. Both 1 and 2
D. Neither 1 nor 2

Correct Answer : C

Answer Justification :

Both statements given above are correct.

36
Total Marks : 200
Online Prelims TEST - 16 (SUBJECT WISE)
( InsightsIAS Mock Test Series for UPSC Preliminary Exam 2020 ) Mark Scored : 0

Tipu Sultan commissioned a military manual called Fathul Mujahidin. The manual explains
the operation of Mysore rockets.

Tipu Sultan laid the foundation for the construction of the Kannambadi dam (present-
day Krishna Raja Sagara or KRS dam) on the Kaveri river, as attested by an extant stone
plaque bearing his name, but was unable to begin the construction. The dam was later built and
opened in 1938. It is a major source of drinking water for the people of Mysore and Bangalore.

60 With reference to the British Policy towards Princely States, consider the following statements:
1. Under the Policy of Ring of Fence the East India Company undertook to organize Awadh’s defence
to safeguard Bengal’s security.
2. The Butler Committee (1927) was set up to examine the nature of relationship between the Princely
states and British Government.

Which of the statements given above is/are correct?


A. 1 only
B. 2 only
C. Both 1 and 2
D. Neither 1 nor 2

Correct Answer : C

Answer Justification :

Both statements given above are correct.

Policy of Ring of Fence was reflected in Warren Hastings’ wars against the Marathas and
Mysore, and aimed at creating buffer zones to defend the Company’s frontiers. The main
threat was from the Marathas and Afghan invaders (the Company undertook to organize
Awadh’s defence to safeguard Bengal’s security).

The Butler Committee (1927) was set up to examine the nature of relationship between the
states and Government.

61 Who among the following social reformer gave the title of Mahatma to Jyotirao phule?

A. Dayanand Saraswati
B. M.K. Gandhi
C. Vithalrao Krishnaji Vandekar
D. Swami Vivekananda

Correct Answer : C

Answer Justification :

37
Total Marks : 200
Online Prelims TEST - 16 (SUBJECT WISE)
( InsightsIAS Mock Test Series for UPSC Preliminary Exam 2020 ) Mark Scored : 0

Mahatma Jyotiba Phule was a visionary much ahead of his times. A pioneer in the field of women’s
education, he is the one who opened the first ever girls’ school in India.

Highly distressed by the malpractices prevalent due to the caste system, he formed the Satya
Shodhak Samaj with the aim of protecting the so-called lower castes from exploitation and
injustices. His unrelenting efforts to eradicate social inequalities earned him the title of “Father of
Indian Social Revolution”.

The great reformer from Mumbai, Rao Bahadur Vithalrao Krishnaji Vandekar gave Jyotirao
Phule the title of “Mahatma” in May 1888 in recognition of his selfless services to humanity.

Hence, option (c) is correct.

62 Which among the following events are associated with the tenure of Lord Ripon?
1. Ilbert Bill controversy
2. Repeal of the Vernacular Press Act
3. Appointment of Hunter Commission
4. Government resolution on local self-government

Select the correct answer using the codes given below:


A. 2 and 4 only
B. 1, 2 and 3 only
C. 1, 3 and 4 only
D. 1, 2, 3 and 4

Correct Answer : D

Answer Justification :

All events given above took place during the tenure Lord Rippon.

Notable works during the tenure of Lord Ripon:

First Factory Act (1881)

Repeal of the Vernacular Press Act (1882)

Ilbert Bill controversy (1883)

Government resolution on local self-government (1882)

Appointment of Education Commission under Sir William Hunter

38
Total Marks : 200
Online Prelims TEST - 16 (SUBJECT WISE)
( InsightsIAS Mock Test Series for UPSC Preliminary Exam 2020 ) Mark Scored : 0

63 What was the reason for awarding the Kaisar – I – Hind medal to Mahatma Gandhi?

A. For successful Gandhi – Irwin Pact


B. Foundation of Indian Ambulance Core during Boers War
C. Foundation of Natal Indian Congress in South Africa
D. Supporting British in First World War on ideological basis

Correct Answer : B

Answer Justification :

The most famous recipient of Kaisar-i-Hind is Mahatma Gandhi, who was awarded the Kaisar-i-
Hind in 1915 by The Lord Hardinge of Penshurst for his contribution to ambulance services
in South Africa. Gandhi returned the medal in 1920 as part of the national campaign
protesting the Jallianwala Bagh massacre and in support of the Khilafat Movement.

Hence, option (b) is correct.

64 Consider the following statements:


1. Iyothee Thass founded the Sadhu Jana Paripalana Sangham (SJPS).
2. Ayyankali established the Advaidananda Sabha.

Which of the statements given above is/are correct?


A. 1 only
B. 2 only
C. Both 1 and 2
D. Neither 1 nor 2

Correct Answer : D

Answer Justification :

Both statements given above are incorrect.

Ayyankali was a social reformer who worked for the advancement of those people in the princely
state of Travancore, British India, who were treated as untouchables. Ayyankali founded
the Sadhu Jana Paripalana Sangham (SJPS) (Association for the Protection of the Poor) which
campaigned for access to schools and raised funds to set up Pulayar-operated schools. C. Iyothee
Thass was a prominent Tamil Activist and anti-caste activist and a practitioner of Siddha medicine.
In 1876, Thass established the Advaidananda Sabha and launched a magazine called Dravida
Pandian in collaboration with Rev. John Rathinam.

65 With reference to the ‘Asiatic Society of Bengal’, consider the following statements:
1. It was established during the rule of Lord Warren Hastings.
2. Dr. Rajendra Lala Mitra was the first Indian President of Asiatic Society

39
Total Marks : 200
Online Prelims TEST - 16 (SUBJECT WISE)
( InsightsIAS Mock Test Series for UPSC Preliminary Exam 2020 ) Mark Scored : 0

3. One of the main objective behind setting up the institute was to promote the study of Indian
languages and scriptures.

Which of the statements given above is/are correct?


A. 1 and 2 only
B. 3 only
C. 2 and 3 only
D. 1, 2 and 3

Correct Answer : D

Answer Justification :

All the statements given above are correct.

Asiatic Society of Bengal, scholarly society founded on Jan. 15, 1784, by Sir William Jones, a British
lawyer and Orientalist, to encourage Oriental studies. At its founding, Jones delivered the first of a
famous series of discourses.

An outstanding scholar from the University of Oxford, Jones arrived in Calcutta (now Kolkata) on
Sept. 25, 1783, as a supreme court judge. The society was founded shortly after his arrival. The
Asiatic Society had the support and encouragement of Warren Hastings, the governor-
general (1772–85) of Bengal, though he declined its presidency.

Until Jones’s death (1794) it was the vehicle for his ideas about the importance of
Hindu culture and learning and about the vital role of Sanskrit in the Aryan languages.
Indians were first admitted as members in 1829.

Headquarters are in Kolkata. The society owns an art collection that includes paintings by Peter
Paul Rubens and Joshua Reynolds. The society’s library contains some 100,000 general volumes,
and its Sanskrit section has more than 27,000 books, manuscripts, prints, coins, and
engravings. The Journal of the Asiatic Society of Bengal is published regularly.

Dr. Rajendra Lala Mitra was the first Indian President of Asiatic Society, who was appointed
in 1885.

66 Consider the following statements:


1. Charles Wilkins translated Bhagwat Gita into English
2. A Sanskrit College was established at Benares during the tenure of Lord Cornwallis.

Which of the statements given above is/are correct?


A. 1 only
B. 2 only
C. Both 1 and 2
D. Neither 1 nor 2

40
Total Marks : 200
Online Prelims TEST - 16 (SUBJECT WISE)
( InsightsIAS Mock Test Series for UPSC Preliminary Exam 2020 ) Mark Scored : 0

Correct Answer : C

Answer Justification :

Both statements given above are correct.

Sir Charles Wilkins was an English typographer and Orientalist, and founding member of The
Asiatic Society. He is notable as the first translator of Bhagavad Gita into English, and as
the creator, alongside Panchanan Karmakar, of the first Bengali typeface.

A Sanskrit College was established at Benares by Jonathan Duncan during the tenure of
Lord Cornwallis in 1791.

67 Namdeo Dhasal is mainly associated with:

A. Workers movement
B. Peasant Movement
C. Tribal movement
D. Dalit movement

Correct Answer : D

Answer Justification :

Namdeo Laxman Dhasal was a Marathi poet, writer and Dalit activist from Maharashtra, India.
Following the example of the American Black Panther movement, he founded the Dalit
Panther movement with friends in 1972. Hence, option (d) is correct.

68 Consider the following pairs:


Social Movement: Region:
1. The Satnami Movement Chhattisgarh
2. Adi Dharma Movement Maharashtra
3. Mahar Movement Punjab

Which of the pairs given above is correctly matched?


A. 1 only
B. 2 and 3 only
C. 1 and 3 only
D. None

Correct Answer : A

Answer Justification :

41
Total Marks : 200
Online Prelims TEST - 16 (SUBJECT WISE)
( InsightsIAS Mock Test Series for UPSC Preliminary Exam 2020 ) Mark Scored : 0

Dalit movements and the meaning of identity, there has been a common quest for equality, self-
dignity and eradication of untouchability.

This can be seen in the Satnami Movement of the Chamars in the Chattisgarh plains in
eastern MP, Adi Dharma Movement in Punjab, the Mahar Movement in Maharashtra, the
socio-political mobilisation among the Jatavas of Agra and the Anti –Brahman Movement in south
India.

Hence, option (a) is correct.

69 With reference to Tej Bahadur Sapru, consider the following statements:


1. He was a law member of the Viceroy’s Council.
2. He had participated in all three Round Table Conference sessions
3. He was also one of the main lawyers engaged to defend captured soldiers of the Indian National
Army.

Which of the statements given above is/are correct?


A. 1 only
B. 2 and 3 only
C. 3 only
D. 1, 2 and 3

Correct Answer : D

Answer Justification :

All the statements given above are correct.

Sir Tej Bahadur Sapru, was a prominent Indian freedom fighter, lawyer and politician. He was a key
figure in India's struggle for independence, helping draft the Indian Constitution. He was the leader
of the Liberal party in British-ruled India.

He was knighted in 1922. He was a member of the United Provinces (now Uttar Pradesh)
Legislative Council (1913–16) and of the Imperial Legislative Council (1916–20), a law member of
the Viceroy’s Council (1920–23), and a delegate to the three Round Table Conference
sessions in London (1930–32) concerning the government of India. His mediation helped to
bring about the Gandhi-Irwin Pact (1931), by which the Indian nationalist leader Mohandas K.
Gandhi terminated a civil disobedience campaign and was allowed to attend the second session of
the Round Table Conference. Sapru was also in part responsible for the Poona Pact modifying the
British plan for a separate electorate of Hindu untouchables

He was also one of the main lawyers engaged to defend captured soldiers of the Indian National
Army.

70 Consider the following statements:


1. The Bombay Textile strike of 1982, was led by George Fernandes.

42
Total Marks : 200
Online Prelims TEST - 16 (SUBJECT WISE)
( InsightsIAS Mock Test Series for UPSC Preliminary Exam 2020 ) Mark Scored : 0

2. The railway strike of 1974 was led by Dr. Datta Samant.

Which of the statements given above is/are correct?


A. 1 only
B. 2 only
C. Both 1 and 2
D. Neither 1 nor 2

Correct Answer : D

Answer Justification :

Both statements given above are incorrect.

The Great Bombay textile strike was a textile strike called on 18 January 1982 by the mill
workers of Bombay under trade union leader Dutta Samant. The purpose of the strike was to
obtain bonus and increase in wages.Nearly 250,000 workers of 65 textile mills went on strike in
Bombay.

1974 railway strike in India was a major strike by the workers of Indian Railways in 1974. The
strike lasted from 8 to 27 May 1974. The 20 day strike by 1.7 million workers is the largest
recorded industrial action in the world. It was led by George Fernandes.

71 Consider the following statements:


1. When the British took over Indian states, towns like Thanjavur, Dhaka, and Murhidabad lost their
courts.
2. In India, the initial impact of the British industrialization led to more people moving into
manufacturing industries.

Which of the statements given above is/are correct?


A. 1 only
B. 2 only
C. Both 1 and 2
D. Neither 1 nor 2

Correct Answer : A

Answer Justification :

In India the impact of the very same British industrialization led to deindustrialization in some
sectors. When the British took over Indian states, towns like Thanjavur, Dhaka, and
Murhidabad lost their courts and, therefore, some of their artisans and court gentry. From the
end of the 19th century, with the installation of mechanized factory industries, some towns became
much more heavily populated. Hence statement 1 is correct.

Unlike Britain where the impact of industrialization led to more people moving into urban areas, in

43
Total Marks : 200
Online Prelims TEST - 16 (SUBJECT WISE)
( InsightsIAS Mock Test Series for UPSC Preliminary Exam 2020 ) Mark Scored : 0

India the initial impact of the same British industrialization led to more people moving into
agriculture. The Census of India Report shows this clearly. Hence statement 2 in incorrect

72 Consider the following statements:


1. In 1938, a National Planning Committee with K.T. Shah as the Chairman and Jawaharlal Nehru as
the general editor was set up by the Indian National Congress.
2. Bhilai steel was set up with the help of the Britain in 1955.
3. Rourkela Steel Plant (RSP) was set up with West German collaboration in 1960s.

Which of the statements given above is/are correct?


A. 1 and 2 only
B. 3 only
C. 1, 2 and 3
D. 1 only

Correct Answer : B

Answer Justification :

In 1938, a National Planning Committee with Jawaharlal Nehru as the Chairman and K.T. Shah
as the general editor was set up by the Indian National Congress. Hence, statement 1 is incorrect.

The Bhilai Steel Plant (BSP), located in Bhilai, in the Indian state of Chhattisgarh, is India's first and
main producer of steel rails, as well as a major producer of wide steel plates and other steel
products. The plant also produces steel and markets various chemical by-products from its coke
ovens and coal chemical plant. It was set up with the help of the USSR in 1955. Hence
statement 2 is incorrect.

Rourkela Steel Plant (RSP), in Rourkela, Odisha is the first integrated steel plant in the public
sector in India. It was set up with West German collaboration with an installed capacity of 1
million tonnes in the 1960s. It is operated by Steel Authority of India. Hence statement 3 is
correct.

73 With reference to the social reformers of India, consider the following statements:
⌰〰〰
1. Pandita Ramabai founded Arya Mahila Samaj.
2. Pandita Ramabai wrote the book ‘High-Caste Hindu Woman’ .
3. Kandukiri Viresalingam started the a journal called Viveka Vardhini .

Which of the statements given above is/are correct?


A. 1 and 2 only
B. 2 and 3 only
C. 1 and 3 only
D. 1, 2 and 3

44
Total Marks : 200
Online Prelims TEST - 16 (SUBJECT WISE)
( InsightsIAS Mock Test Series for UPSC Preliminary Exam 2020 ) Mark Scored : 0

Correct Answer : D

Answer Justification :

All the statements given above are correct.

Pandita Ramabai Sarasvati (23 April 1858 – 5 April 1922) was an Indian social reformer, a pioneer
in the education and emancipation of women in India. She was the first woman to be accorded the
titles of Pandita as a Sanskrit scholar and Sarasvati after being examined by the faculty of the
University of Calcutta. She was one of the 10 women delegates of the Congress session of 1889

Pandita Ramabai founded Arya Mahila Samaj. The purpose of the society was to promote the
cause of women's education and deliverance from the oppression of child marriage. When in 1882 a
commission was appointed by Government of India to look into education, Ramabai gave evidence
before it.

Pandita Ramabai wrote the book ‘High-Caste Hindu Woman’. This was also the first book that
she wrote in English. Ramabai dedicated this book to Dr. Joshi, The High-Caste Hindu Woman-to be
specific a Brahmin woman which showed the darkest aspects of the life of Hindu women, including
child brides and child widows.

Rao Bahadur Kandukuri Veeresalingam Pantulu (16 April 1848 – 27 May 1919) was a
social reformer and writer of Andhra Pradesh, India One of the greatest reforms of
Veeresalingam was to promote women's education, which was considered to be a taboo in those
days. In 1876, he started a journal called Viveka Vardhini and published articles about
women's issues of that era. The magazine was initially printed at Chennai (then Madras), but
with his writings gaining popularity, he established his own press at Rajahmundry.

74 Consider the following statements:


1. Ishwar Chandra Vidyasagar played a major role in legalizing the widow remarriage.
2. Jyotiba Phule formed Arya Samaj to focus on rights of depressed groups.
3. In 1935, Jahanara Shah Nawas founded the Punjab Provincial Women's Muslim League.

Which of the statements given above is/are correct?


A. 1 only
B. 1 and 3 only
C. 2 and 3 only
D. 1, 2 and 3

Correct Answer : B

Answer Justification :

Ishwar Chandra Vidyasagar played a major role in legalizing the widow remarriage, which
led to enactment of Widow remarriage Act, 1956. Hence, statement 1 is correct.

Jyotiba Phule formed Satya Shodak Samaj to focus on rights of depressed groups. Hence,

45
Total Marks : 200
Online Prelims TEST - 16 (SUBJECT WISE)
( InsightsIAS Mock Test Series for UPSC Preliminary Exam 2020 ) Mark Scored : 0

statement 2 is incorrect.

Begum Jahanara Shahnawaz was a politician and Muslim League activist. In 1935, she
founded the Punjab Provincial Women's Muslim League . Hence, statement 3 is correct.

75 Consider the following statements:


1. Raja Ravi Varma completely discarded techniques of the European academic art in his paintings.
2. O. Chandu Menon authored the famous novel Indulekha in Malayalam language.
3. Bharat Mata Painting was one of the notable art works of Abanindranath Tagore.

Which of the statements given above is/are correct?


A. 1 only
B. 2 and 3 only
C. 1 and 3 only
D. 3 only

Correct Answer : B

Answer Justification :

While continuing the tradition and aesthetics of Indian art, Ravi Verma’s paintings employed the
latest European academic art techniques of the day. Hence, statement 1 is incorrect.

O. Chandu Menon authored the famous novel Indulekha in Malayalam language. Hence, statement
2 is correct.

Bharat Mata Painting was one of the notable art works of Abanindranath Tagore. Hence,
statement 3 is correct.

76 Which of the following fundamental rights was/were proposed in Karachi Resolution of Indian
National Congress in 1931?
1. Free compulsory education from primary to university level.
2. Capital punishment to be abolished.
3. Religious neutrality of State.
4. Abolition of Salt tax.

Select the correct answer using the code given below:


A. 1, 2 and 3 only
B. 3 and 4 only
C. 2 and 4 only
D. 2, 3 and 4 only

Correct Answer : D

Answer Justification :

46
Total Marks : 200
Online Prelims TEST - 16 (SUBJECT WISE)
( InsightsIAS Mock Test Series for UPSC Preliminary Exam 2020 ) Mark Scored : 0

Karachi Resolution of Indian National Congress in 1931 provided for Free compulsory education till
primary level. Hence, statement 1 is incorrect.

Other three provisions are the proposed under Karachi Resolution of Indian National Congress in
1931.

77 With reference to the Chipko movement, consider the following statement:


1. It is a forest conservation movement.
2. In 1987, the Chipko movement was awarded the Right Livelihood Award.

Which of the statements given above is/are correct?


A. 1 only
B. 2 only
C. Both 1 and 2
D. Neither 1 nor 2

Correct Answer : C

Answer Justification :

Both statements are correct.

The Chipko movement, or Chipko Andolan, was a forest conservation movement in India. It
began in 1974 in Reni village of Chamoli district, Uttarakhand and went on to become a rallying
point for many future environmental movements all over the world.

In 1987, the Chipko movement was awarded the Right Livelihood Award.

78 Which of the following state government launched the land reform movement ‘Operation Barga’?

A. West Bengal
B. Maharashtra
C. Odisha
D. Andhra Pradesh

Correct Answer : A

Answer Justification :

Operation Barga was a land reform movement throughout rural West Bengal for recording the
names of sharecroppers while avoiding the time-consuming method of recording through the
settlement machinery.

Hence, option (a) is correct.

47
Total Marks : 200
Online Prelims TEST - 16 (SUBJECT WISE)
( InsightsIAS Mock Test Series for UPSC Preliminary Exam 2020 ) Mark Scored : 0

79 Consider the following statements:


1. Tebhaga movement was a struggle of sharecroppers in Maharashtra for two thirds share of their
produce instead of the customary half.
2. The Naxalite movement started from the region of Naxalbari (1967) in Bengal.
3. The Telangana movement (1946-51) directed against the feudal conditions in the princely state of
Hyderabad.

Which of the statements given above is/are correct?


A. 1 only
B. 2 and 3 only
C. 1 and 2 only
D. 1, 2 and 3

Correct Answer : B

Answer Justification :

At the time of Independence we had the two most classical cases of peasant
movements, namely the Tebhaga movement (1946-7) and the Telangana
movement (1946-51). The first was a struggle of sharecroppers in Bengal in North Bihar for two
thirds share of their produce instead of the customary half. It had the support of the Kisan Sabha
and the Communist Party of India (CPI). Hence, statement 1 is incorrect.

The second, directed against the feudal conditions in the princely state of Hyderabad and was led
by the CPI. Hence, statement 3 is correct.

The Naxalite movement started from the region of Naxalbari (1967) in Bengal. Hence, statement
2 is correct.

80 Consider the following statements:


1. Stree Purush Tulana (or Comparison of Men and Women) was written by Tarabai Shinde.
2. Sultana’s Dream was written Begum Rokeya Sakhawat Hossain .

Which of the statements given above is/are correct?


A. 1 only
B. 2 only
C. Both 1 and 2
D. Neither 1 nor 2

Correct Answer : C

Answer Justification :

Stree Purush Tulana (or Comparison of Men and Women) was written by a
Maharashtrian housewife, Tarabai Shinde, as a protest against the double

48
Total Marks : 200
Online Prelims TEST - 16 (SUBJECT WISE)
( InsightsIAS Mock Test Series for UPSC Preliminary Exam 2020 ) Mark Scored : 0

standards of a male dominated society. Hence, statement 1 is correct.

Rokeya Sakhawat Hossain, commonly known as Begum Rokeya, was a Bengali feminist thinker,
educator and political activist from British India. She is widely regarded as a pioneer of women's
liberation in South Asia.

Her major works include Matichur (A String of Sweet Pearls, 1904 and 1922), a collection of
essays in two volumes expressing her feminist thoughts; Sultana's Dream (1905), a feminist
science fiction novella set in Ladyland ruled by women; Padmarag ("Essence of the Lotus", 1924)
depicting the difficulties faced by Bengali wives and Abarodhbasini (The Confined Women, 1931),
a spirited attack on the extreme forms of purdah that endangered women's lives and self-image.
Hence, statement 2 is correct.

81 With reference to the SAANS campaign, consider the following statements:


1. It was launched by the Union Ministry of Health and Family Welfare.
2. It aimed at reducing child mortality due to tuberculosis (TB).
3. Under the campaign, Pulse Oximeter will be used at the Health and Wellness Centre for
identification of low oxygen levels in the blood.

Which of the statements given above is/are correct?


A. 1 and 3 only
B. 3 only
C. 2 only
D. 1, 2 and 3

Correct Answer : A

Answer Justification :

During the sixth National Summit on Good, Replicable Practices and Innovations in Public
Health Care Systems SAANS (“Social Awareness and Action to Neutralize Pneumonia
Successfully” ) was launched by the Union Ministry of Health and Family Welfare. Hence,
statement 1 is correct.

This campaign will mobilize people's for protecting and preventing children from pneumonia;
and health personnel, governments & other stakeholders to prioritize treatment towards the control
of Pneumonia — one of the most fatal childhood illnesses. Hence, statement 2 is incorrect.

Under the SAANS campaign, new interventions have been included like a child suffering from
Pneumonia will be treated with pre-referral dose of Amoxicillin by ASHA; Pulse Oximeter will be
used at the Health and Wellness Centre for identification of low oxygen levels in the blood
and if required treated by use of Oxygen. The initiative also aims to create mass awareness
about the most effective solutions for pneumonia prevention like breast feeding, age appropriate
complementary feeding, immunization, good quality air etc. Hence, statement 3 is correct.

https://pib.gov.in/PressReleasePage.aspx?PRID=1591826

49
Total Marks : 200
Online Prelims TEST - 16 (SUBJECT WISE)
( InsightsIAS Mock Test Series for UPSC Preliminary Exam 2020 ) Mark Scored : 0

82 Consider the following statements:


1. SeeTB is a new vaccine developed by Indian researchers to treat Multi-Drug Resistant Tuberculosis
(MDR-TB).
2. The World Health Organization (WHO) has aimed at eliminating Tuberculosis by 2025.
3. Tuberculosis is a bacterial disease.

Which of the statements given above is/are correct?


A. 2 and 3 only
B. 3 only
C. 1 and 2 only
D. 1, 2 and 3

Correct Answer : B

Answer Justification :

Tuberculosis (TB) is a disease caused by bacteria called Mycobacterium tuberculosis. The


bacteria usually attack the lungs, but they can also damage other parts of the body. TB spreads
through the air when a person with TB of the lungs or throat coughs, sneezes, or talks. Hence,
statement 3 is correct.

Indian researchers have developed a small device named SeeTB, to detect Tuberculosis. It is a
small device that can be attached to a simple optical microscope to convert it into a
fluorescence microscope, thus enabling better TB diagnosis at the point of care. Hence,
statement 1 is incorrect.

The World Health Organization has aimed at eliminating TB by 2035, and the Indian
government has vouched to do this by 2025. Hence, statement 2 is incorrect.

https://www.thehindu.com/sci-tech/science/seetb-new-diagnostic-tool-for-detecting-tuberculosis/arti
cle29993209.ece

83 Consider the following countries:


1. India
2. Japan
3. South Korea
4. New Zealand

Which of the countries given above is/are part of the ASEAN Defence Ministers’ Meeting Plus
(ADMM-Plus)?
A. 1 and 3 only
B. 1, 2 and 4 only
C. 3 only
D. 1, 2, 3 and 4

50
Total Marks : 200
Online Prelims TEST - 16 (SUBJECT WISE)
( InsightsIAS Mock Test Series for UPSC Preliminary Exam 2020 ) Mark Scored : 0

Correct Answer : D

Answer Justification :

All the countries given above are part of the ASEAN Defence Ministers’ Meeting Plus
(ADMM-Plus).

Consistent with the ADMM guiding principles of open and outward looking, the 2ndADMM in
Singapore in 2007 adopted the Concept Paper to establish the ADMM-Plus.The ADMM-Plus is a
platform for ASEAN and its eight Dialogue Partners to strengthen security and defence cooperation
for peace, stability, and development in the region.

The ADMM-Plus countries include ten ASEAN Member States, namely, Brunei Darussalam,
Cambodia, Indonesia, Lao PDR, Malaysia, Myanmar, the Philippines, Singapore, Thailand
and Viet Nam, and eight Plus countries, namely Australia, China, India, Japan, New
Zealand, ROK, Russian Federation, and the United States.

https://www.thehindu.com/news/national/rajnath-singh-to-attend-the-admm-plus-meeting-in-bangko
k-today/article29994626.ece

https://admm.asean.org/index.php/about-admm/about-admm-plus.html

84 With reference to the Pradhan Mantri Matru Vandana Yojana (PMMVY), consider the following
statements:
1. It is a centrally sponsored scheme.
2. For the case of Miscarriage/Still Birth, a beneficiary is eligible to receive benefits under the scheme
only once.
3. Cash incentives of Rs. 10000 will be provided to the beneficiaries in three installments.

Which of the statements given above is/are correct?


A. 1 only
B. 1 and 2 only
C. 2 and 3 only
D. 1, 2 and 3

Correct Answer : B

Answer Justification :

PMMVY is a Centrally Sponsored Scheme of the Government for pregnant women and lactating
mothers. Hence, statement 1 is correct.

Case of Miscarriage/Still Birth :

A beneficiary is eligible to receive benefits under the scheme only once. Hence,
statement 2 is correct.

51
Total Marks : 200
Online Prelims TEST - 16 (SUBJECT WISE)
( InsightsIAS Mock Test Series for UPSC Preliminary Exam 2020 ) Mark Scored : 0

In case of miscarriage/still birth, the beneficiary would be eligible to claim the remaining
instalment(s) in event of any future pregnancy.

Thus, after receiving the 1st instalment, if the beneficiary has a miscarriage, she would only
be eligible for receiving 2nd and 3rd instalment in event of future pregnancy subject to
fulfilment of eligibility criterion and conditionalities of the scheme. Similarly, if the
beneficiary has a miscarriage or still birth after receiving 1 st and 2nd instalments, she would
only be eligible for receiving 3rd instalment in event of future pregnancy subject to fulfilment
of eligibility criterion and conditionalities of the scheme.

Cash incentive of Rs 5000 in three instalments i.e. first instalment of Rs 1000/ - on early
registration of pregnancy at the Anganwadi Centre (AWC) / approved Health facility as may be
identified by the respective administering State / UT, second instalment of Rs 2000/ - after six
months of pregnancy on receiving at least one ante-natal check-up (ANC) and third instalment of Rs
2000/ - after child birth is registered and the child has received the first cycle of BCG, OPV, DPT
and Hepatitis - B, or its equivalent/ substitute. Hence, statement 3 is incorrect.

The eligible beneficiaries would receive the incentive given under the Janani Suraksha Yojana (JSY)
for Institutional delivery and the incentive received under JSY would be accounted towards
maternity benefits so that on an average a woman gets Rs 6000 / - .

https://www.thehindu.com/news/national/maternity-scheme-reaches-only-one-third-of-beneficiaries/a
rticle30009783.ece

http://vikaspedia.in/social-welfare/women-and-child-development/women-development-1/pradhan-m
antri-matru-vandana-yojana

85 Consider the following statements:


1. The Kimberley Process is an international certification scheme that regulates trade in rough
diamonds.
2. India is yet to become the member of Kimberley Process.

Which of the statements given above is/are correct?


A. 1 only
B. 2 only
C. Both 1 and 2
D. Neither 1 nor 2

Correct Answer : A

Answer Justification :

The Kimberley Process (KP) is a commitment to remove conflict diamonds from the global
supply chain. Today, participants actively prevent 99.8% of the worldwide trade. Uniting
governments, civil society and the wider industry, the Kimberley Process (KP) defines conflict

52
Total Marks : 200
Online Prelims TEST - 16 (SUBJECT WISE)
( InsightsIAS Mock Test Series for UPSC Preliminary Exam 2020 ) Mark Scored : 0

diamonds as: ‘rough diamonds used to finance wars against governments’ - around the
world. Hence, statement 1 is correct.

India is the present Kimberley Process (KP) Chair. The plenary is being hosted in New Delhi in
November 201 9. Hence, statement 2 is incorrect.

https://pib.nic.in/PressReleasePage.aspx?PRID=1591975

https://www.kimberleyprocess.com/en/what-kp

86 Consider the following statements:


1. Dam Rehabilitation and Improvement Project was launched in 2012 by Central Water Commission
(CWC).
2. Dam Health And Rehabilitation Monitoring Application (DHARMA) is a bespoke web-based software
package to support the effective collection and management of Dam Safety data in respect of all
large dams of India.

Which of the statements given above is/are correct?


A. 1 only
B. 2 only
C. Both 1 and 2
D. Neither 1 nor 2

Correct Answer : C

Answer Justification :

Both statements given above are correct.

India ranks third globally with 5334 large dams in operation and about 411 are under construction.
In addition, there are several thousand smaller dams. These dams are vital for ensuring the water
security of the Country; and these also constitute a major responsibility in terms of asset
management and safety. In April 2012, Ministry of Water Resources, River Development &
Ganga Rejuvenation through Central Water Commission with an objective to improve safety
and operational performance of selected dams, along with institutional strengthening with system
wide management approach, embarked upon the six year Dam Rehabilitation and Improvement
Project (DRIP) with World Bank assistance at a cost of INR 2100 Crore (US$M 437.5).

The project originally envisaged the rehabilitation and improvement of 223 dam projects in four
states namely, Kerala, Madhya Pradesh, Odisha, and Tamil Nadu. Later Karnataka, Uttarakhand
(UJVNL) and Damodar Valley Corporation (DVC) joined the DRIP, and number of dams in DRIP
portfolio increased to 257; due to the addition/deletion of few dams during implementation by
partner agencies, presently 198 dam projects are being rehabilitated.

Dam Health And Rehabilitation Monitoring Application (DHARMA) is a bespoke web-based


software package to support the effective collection and management of Dam Safety data in respect
of all large dams of India. The software is designed for users at Central, State and Dam level, with

53
Total Marks : 200
Online Prelims TEST - 16 (SUBJECT WISE)
( InsightsIAS Mock Test Series for UPSC Preliminary Exam 2020 ) Mark Scored : 0

user permission rights governed by their respective licenses.

https://www.thehindu.com/news/national/andhra-pradesh/srisailam-dam-in-need-of-urgent-maintena
nce/article30020482.ece

https://damsafety.in/index.php?lang=en&page=homeinfo&origin=front-end&tp=1&rn=1

https://damsafety.in/dharma/Home1/index.php

87 With reference to the Insolvency and Bankruptcy Board of India (IBBI) , consider the following
statements:
1. It was established under the Company Act, 2013.
2. It has regulatory oversight over the Insolvency Professionals, Insolvency Professional Agencies,
Insolvency Professional Entities and Information Utilities.
3. It has been designated as the ‘Authority’ under the Companies (Registered Valuers and Valuation
Rules), 2017 for regulation and development of the profession of valuers in the country.

Which of the statements given above is/are correct?


A. 1 only
B. 1 and 2 only
C. 2 and 3 only
D. 1, 2 and 3

Correct Answer : C

Answer Justification :

The Insolvency and Bankruptcy Board of India was established on 1st October, 2016 under the
Insolvency and Bankruptcy Code, 2016 (Code). Hence, statement 1 is incorrect.

It is a unique regulator: regulates a profession as well as processes. It has regulatory oversight


over the Insolvency Professionals, Insolvency Professional Agencies, Insolvency
Professional Entities and Information Utilities. Hence, statement 2 is correct.

It has recently been tasked to promote the development of, and regulate, the working and practices
of, insolvency professionals, insolvency professional agencies and information utilities and other
institutions, in furtherance of the purposes of the Code. It has also been designated as the
‘Authority’ under the Companies (Registered Valuers and Valuation Rules), 2017 for
regulation and development of the profession of valuers in the country. Hence, statement 3
is correct.

https://www.thehindu.com/opinion/editorial/a-precedent-on-apex-court-ruling-on-essar-steel/article3
0009322.ece

https://ibbi.gov.in/about

88 With reference to the Tobacco Board, consider the following statements:

54
Total Marks : 200
Online Prelims TEST - 16 (SUBJECT WISE)
( InsightsIAS Mock Test Series for UPSC Preliminary Exam 2020 ) Mark Scored : 0

1. It is a statutory body established under Ministry of Agriculture and Farmers’ Welfare.


2. One of the functions of the board is to regulate the production and curing of Virginia Tobacco with
regard to the demand in India and abroad.
3. Recently, it has been awarded the Golden Leaf Award.

Which of the statements given above is/are correct?


A. 1 only
B. 2 and 3 only
C. 1 and 3 only
D. 1, 2 and 3

Correct Answer : B

Answer Justification :

Recognizing the need to regulate production, promote overseas marketing and control recurring
instances of imbalances in supply and demand, which lead to market problems, the Government of
India under the Tobacco Board Act of 1975, established the Tobacco Board, in place of the
Tobacco Export Promotion Council. The Board came into existence from 1-1-1976 and opened its
headquarters at Guntur in Andhra Pradesh, India. It works under the Ministry of Commerce and
Industry. Hence, statement 1 is incorrect.

The various activities of the Board outlined in the Act for the promotion of the industry are-

Regulating the production and curing of Virginia Tobacco with regard to the demand
in India and abroad. Hence, statement 2 is correct.

Constant monitoring of the Virginia tobacco market, both in India and abroad and ensuring
fair and remunerative price to the growers and reducing wide fluctuations in the prices of the
commodity etc

Tobacco Board of India, Guntur, has been awarded the Golden Leaf Award at Tab Expo 2019
event in Amsterdam, Netherlands. Hence, statement 3 is correct.

https://www.thehindu.com/news/cities/Vijayawada/tobacco-board-bags-golden-leaf-award/article299
87096.ece

https://tobaccoboard.com/bactivities.php#

89 With reference to the recently inaugurated Sisseri River Bridge, consider the following statements:
1. It will provide connectivity between Dibang Valley and Siang in Arunachal Pradesh.
2. It was constructed by Project Brahmank of Border Roads Organization (BRO).

Which of the following statements given above is/are correct?


A. 1 only

55
Total Marks : 200
Online Prelims TEST - 16 (SUBJECT WISE)
( InsightsIAS Mock Test Series for UPSC Preliminary Exam 2020 ) Mark Scored : 0

B. 2 only
C. Both 1 and 2
D. Neither 1 nor 2

Correct Answer : C

Answer Justification :

Both statements given above are correct.

Defence Minister Rajnath Singh inaugurated the Sisseri River bridge at Lower Dibang Valley in
Arunachal Pradesh. The 200-metre long bridge between Jonai-Pasighat-Ranaghat-Roing
road will provide connectivity between Dibang Valley and Siang to meet the long-pending
demand of the people of Arunachal Pradesh as it would cut down the travel time from Pasighat to
Roing by about five hours.

The Sisseri River bridge provides connectivity to Tinsukia via Dhola-Sadia bridge. It was
constructed by Project Brahmank of Border Roads Organisation (BRO). This bridge is also
strategically important from military view point and will be a part of Trans Arunachal Highway.

https://pib.gov.in/PressReleasePage.aspx?PRID=1591755

90 Consider the following statements:


1. Jal Jeevan Mission aims to provide tap water to all households by 2022.
2. The Jal Shakti Abhiyan (JSA) is a time-bound, mission-mode water conservation campaign.

Which of the following statements given above is/are correct?


A. 1 only
B. 2 only
C. Both 1 and 2
D. Neither 1 nor 2

Correct Answer : B

Answer Justification :

The big programme announced by the Prime Minister — Jal Jeevan Mission — is to give functional
tap-water to every household in the country by 2024. Hence, statement 1 is incorrect.

The Jal Shakti Abhiyan (JSA) is a time-bound, mission-mode water conservation campaign.
The JSA will run in two Phases: Phase 1 from 1st July to 15th September 2019 for all States and
Union Territories; and Phase 2 from 1st October to 30th November 2019 for States and UTs
receiving the retreating monsoon (Andhra Pradesh, Karnataka, Puducherry and Tamil Nadu).
Hence, statement 2 is correct.

https://pib.gov.in/newsite/PrintRelease.aspx?relid=194516

56
Total Marks : 200
Online Prelims TEST - 16 (SUBJECT WISE)
( InsightsIAS Mock Test Series for UPSC Preliminary Exam 2020 ) Mark Scored : 0

http://vikaspedia.in/events/jal-shakti-abhiyan

91 With reference to the recently announced Bharatiya Poshan Krishi Kosh, consider the following
statements:
1. It is a repository of diverse crops across 128 agro-climatic zones for better nutritional outcomes.
2. It aims at reducing malnutrition through a multi-sectoral results-based framework, including
agriculture, among women and children across the country.
3. Ministry of Women and Child Development has collaborated with Bill and Melinda Gates Foundation
for this project.

Which of the statements given above is/are correct?


A. 1 only
B. 2 and 3 only
C. 1 and 3 only
D. 1, 2 and 3

Correct Answer : D

Answer Justification :

All the statements given above are correct.

Union Government announced Bharatiya Poshan Krishi Kosh which is a repository of diverse
crops across 128 agro-climatic zones for better nutritional outcomes. The Kosh aims at
reducing malnutrition through a multi-sectoral results-based framework, including
agriculture, among women and children across the country.

Ministry of Women and Child Development has collaborated with Bill and Melinda Gates
Foundation for this project. The Bharatiya Poshan Krishi Kosh seeks to promote healthy dietary
practices and tackle under-nutrition in a sustainable manner. The initiative will be built on an
essential understanding of social, behavioural and cultural practices that promote and reinforce
healthy dietary behaviours both at the individual and community level.

http://www.newsonair.com/Main-News-Details.aspx?id=374695

92 With reference to the World Customs Organization, consider the following statements:
1. It is a specialized agency of United Nations.
2. India is a member of the World Customs Organization.

Which of the following statements given above is/are correct?


A. 1 only
B. 2 only
C. Both 1 and 2
D. Neither 1 nor 2

57
Total Marks : 200
Online Prelims TEST - 16 (SUBJECT WISE)
( InsightsIAS Mock Test Series for UPSC Preliminary Exam 2020 ) Mark Scored : 0

Correct Answer : B

Answer Justification :

The World Customs Organization (WCO), established in 1952 as the Customs Co-operation Council
(CCC) is an independent intergovernmental body whose mission is to enhance the effectiveness
and efficiency of Customs administrations. Hence, statement 1 is incorrect.

Today, the WCO represents 183 Customs administrations across the globe that collectively
process approximately 98% of world trade. As the global centre of Customs expertise, the WCO is
the only international organization with competence in Customs matters and can rightly call itself
the voice of the international Customs community.

India is a member of WCO. Hence, statement 2 is correct.

https://aninews.in/news/national/general-news/puducherry-29th-rcp-meeting-of-asia-pacific-customs-
begins20191119013454/

http://www.wcoomd.org/en/about-us/what-is-the-wco.aspx

93 Changi naval base, sometimes seen in news is situated in:

A. Malaysia
B. Vietnam
C. Singapore

58
Total Marks : 200
Online Prelims TEST - 16 (SUBJECT WISE)
( InsightsIAS Mock Test Series for UPSC Preliminary Exam 2020 ) Mark Scored : 0

D. Japan

Correct Answer : C

Answer Justification :

India and Singapore have signed Bilateral Agreement for Navy Cooperation that will allow Indian
Navy ships logistical support, including refuelling at Singapore’s Changi Naval Base located near
disputed South China Sea (SCS). The agreement was announced during second Defence Ministers’
Dialogue between India and Singapore held in New Delhi.

https://economictimes.indiatimes.com/news/defence/rajnath-singh-visits-sembawang-air-base-in-sing
apore/videoshow/72123484.cms

94 Which of the following heritage sites chosen by Geological survey of India are included under
UNESCO Global Geopark Network?
1. St. Mary’s island
2. Manas National park
3. Sunderbans
4. Lonar lake

Select the correct answer using the code given below:


A. 1, 2 and 4 only
B. 1 and 4 only
C. 2 and 3 only
D. None

Correct Answer : B

Answer Justification :

Geological Survey of India has chosen heritage locations in Maharashtra and Karnataka for
UNESCO Global Geopark Network status. The Geopark tag is akin to that of a ‘World Heritage Site’
for historical monuments that can bring India’s famed geological features to the global stage.

The sites chosen are- Lonar Lake in Maharashtra and St. Mary’s Island and Malpe beach in
coastal Karnataka are the GSI’s candidates for UNESCO Global Geopark Network status.

https://www.thehindu.com/news/national/88-million-year-old-isle-and-crater-to-be-geoparks/article2
5105544.ece

95 With reference to the ‘PM-AASHA’ scheme, consider the following statements:


1. It is aimed at ensuring better nutrition and medical facilities for ASHA and Anganwadi workers.
2. Under this scheme the beneficiaries will also get medical insurance.

59
Total Marks : 200
Online Prelims TEST - 16 (SUBJECT WISE)
( InsightsIAS Mock Test Series for UPSC Preliminary Exam 2020 ) Mark Scored : 0

Which of the statements given above is /are correct?


A. 1 only
B. 2 only
C. Both 1 and 2
D. Neither 1 nor 2

Correct Answer : D

Answer Justification :

Both statements given above are incorrect.

Pradhan Mantri Annadata Aay SanraksHan Abhiyan (PM-AASHA) is an umbrella scheme aimed at
ensuring remunerative prices to the farmers for their produce.

http://vikaspedia.in/agriculture/policies-and-schemes/crops-related/pradhan-mantri-annadata-aay-sa
nrakshan-abhiyan

https://www.business-standard.com/article/news-cm/cabinet-decides-to-hike-msp-for-rabi-crops-119
102301119_1.html

96 With reference to Rashtriya Arogya Nidhi (RAN), consider the following statements:
1. It is set up as a not-for-profit company under section 8 of the Company Act, 2013.
2. It provides for financial assistance to patients, living below the poverty line and who are suffering
from major life-threatening diseases.

Which of the statements given above are correct?


A. 1 only
B. 2 only
C. Both 1 and 2
D. Neither 1 nor 2

Correct Answer : B

Answer Justification :

The scheme Rashtriya Arogya Nidhi (RAN) is under implementation in the country.

The Rashtriya Arogya Nidhi (RAN) has been set up vide Resolution No. F-7-2/96-Fin-II dated
13/1/1997 and registered under the Societies Registration Act, 1860, as a Society. Hence,
statement 1 is incorrect.

The RAN was set up to provide financial assistance to patients, living below poverty line and who
are suffering from major life threatening diseases, to receive medical treatment at any of the
super speciality Hospitals/Institutes or other Government hospitals. Hence, statement 2 is
correct.

60
Total Marks : 200
Online Prelims TEST - 16 (SUBJECT WISE)
( InsightsIAS Mock Test Series for UPSC Preliminary Exam 2020 ) Mark Scored : 0

The financial assistance to such patients is released in the form of ‘one-time grant’, which is
released to the Medical Superintendent of the Hospital in which the treatment has been/is being
received.

https://www.ndtv.com/india-news/additional-cover-under-ayushman-bharat-not-allowed-health-minis
try-2134021

http://www.pib.nic.in/Pressreleaseshare.aspx?PRID=1524808

97 Government of India has signed an agreement with which of the following bank to provide a $250-
million loan for the National Rural Economic Transformation Project?

A. New development Bank


B. Asian Infrastructure Investment Bank
C. Asian Development Bank
D. World Bank

Correct Answer : D

Answer Justification :

The World Bank and the Government of India signed a $250 Million Agreement for
the National Rural Economic Transformation Project (NRETP) which will help women in
rural households shift to a new generation of economic initiatives by developing viable
enterprise for farm and non-farm products.

A Key Focus of the Project will be to promote women-owned and women-led farm and non-farm
enterprises across value chains; enable them to build businesses that help them access finance,
markets and networks; and generate employment.

The National Rural Economic Transformation Project (NRETP) is an additional financing to the
$500 million National Rural Livelihoods Project (NRLP) approved by the World Bank in July 2011.

Hence, option (d) is correct.

https://www.business-standard.com/article/news-cm/cabinet-approves-implementation-of-national-ru
ral-economic-transformation-project-119022000240_1.html

98 With reference to the United Nations Office on Drugs and Crime (UNODC), consider the following
statements:
1. UNODC is mandated to assist Member States in their struggle against illicit drugs, crime and
terrorism.
2. It was established in 1997 through a merger between the United Nations Drug Control Programme
and the Centre for International Crime Prevention.

Which of the above given statements is/are correct?


A. 1 only

61
Total Marks : 200
Online Prelims TEST - 16 (SUBJECT WISE)
( InsightsIAS Mock Test Series for UPSC Preliminary Exam 2020 ) Mark Scored : 0

B. 2 only
C. Both 1 and 2
D. Neither 1 nor 2

Correct Answer : C

Answer Justification :

Both statements given above are correct.

UNODC is a global leader in the fight against illicit drugs and international crime. Established in
1997 through a merger between the United Nations Drug Control Programme and the
Centre for International Crime Prevention, UNODC operates in all regions of the world
through an extensive network of field offices. UNODC relies on voluntary contributions,
mainly from Governments, for 90 per cent of its budget.

UNODC is mandated to assist Member States in their struggle against illicit drugs, crime
and terrorism. In the Millennium Declaration, Member States also resolved to intensify efforts to
fight transnational crime in all its dimensions, to redouble the efforts to implement the commitment
to counter the world drug problem and to take concerted action against international terrorism.

The three pillars of the UNODC work programme are:

Field-based technical cooperation projects to enhance the capacity of Member States to


counteract illicit drugs, crime and terrorism

Research and analytical work to increase knowledge and understanding of drugs and crime
issues and expand the evidence base for policy and operational decisions

Normative work to assist States in the ratification and implementation of the relevant
international treaties, the development of domestic legislation on drugs, crime and terrorism,
and the provision of secretariat and substantive services to the treaty-based and governing
bodies

http://www.newsonair.com/News?title=India-calls-for-zero-tolerance-without-%26%2339%3Bdouble
-standards%26%2339%3B-to-combat-terror-crime-nexus&id=374810

https://www.unodc.org/unodc/en/about-unodc/index.html?ref=menutop

99 ‘Idlib Province’, recently seen in news , is located in?

A. Iran
B. Syria
C. Afghanistan

62
Total Marks : 200
Online Prelims TEST - 16 (SUBJECT WISE)
( InsightsIAS Mock Test Series for UPSC Preliminary Exam 2020 ) Mark Scored : 0

D. Iraq

Correct Answer : B

Answer Justification :

Idlib Governorate is one of the 14 governorates (provinces) of Syria. It is situated in


northwestern Syria, bordering Turkey.

https://www.thehindu.com/news/international/syrian-shelling-of-camp-housing-displaced-people-kills
-15/article30031704.ece

100 Which of the following became first State to implement biofuel policy?

A. Rajasthan
B. Karnataka
C. Maharashtra
D. Gujarat

Correct Answer : A

Answer Justification :

Recently, Rajasthan became first state in the country to implement National Policy on Biofuels
unveiled by Union Government in May 2018.

A initiative to provide watermarks and bookmarks free pdfs to you.


Share and Subscribe our telegram channel
@visionpt3652019

https://t.me/visionpt3652019
63

You might also like